Примеры решения систем линейных уравнений метод гаусса: Решение систем линейных уравнений методом Гаусса

Содержание

принцип, теорема и примеры решения задач

Задание. Решить СЛАУ $\left\{\begin{array}{l} 2 x_{1}+x_{2}+x_{3}=2 \\ x_{1}-x_{2}=-2 \\ 3 x_{1}-x_{2}+2 x_{3}=2 \end{array}\right.$ методом Гаусса.

Решение. Выпишем расширенную матрицу системы и при помощи элементарных преобразований над ее строками приведем эту матрицу к ступенчатому виду (прямой ход) и далее выполним обратный ход метода Гаусса (сделаем нули выше главной диагонали). Вначале поменяем первую и вторую строку, чтобы элемент $a_{11}$ равнялся 1 (это мы делаем для упрощения вычислений):

$$\tilde{A}=A \mid B=\left(\begin{array}{rrr|r} 2 & 1 & 1 & 2 \\ 1 & -1 & 0 & -2 \\ 3 & -1 & 2 & 2 \end{array}\right) \sim\left(\begin{array}{rrr|r} 1 & -1 & 0 & -2 \\ 2 & 1 & 1 & 2 \\ 3 & -1 & 2 & 2 \end{array}\right)$$

Далее делаем нули под главной диагональю в первом столбце. Для этого от второй строки отнимаем две первых, от третьей — три первых:

$$\tilde{A} \sim\left(\begin{array}{rrr|r} 1 & -1 & 0 & -2 \\ 0 & 3 & 1 & 6 \\ 0 & 2 & 2 & 8 \end{array}\right)$$

Все элементы третьей строки делим на два (или, что тоже самое, умножаем на $\frac{1}{2}$ ):

$$\tilde{A} \sim\left(\begin{array}{rrr|r} 1 & -1 & 0 & -2 \\ 0 & 3 & 1 & 6 \\ 0 & 1 & 1 & 4 \end{array}\right)$$

Далее делаем нули во втором столбце под главной диагональю, для удобства вычислений поменяем местами вторую и третью строки, чтобы диагональный элемент равнялся 1:

$$\tilde{A} \sim\left(\begin{array}{ccc|c} 1 & -1 & 0 & -2 \\ 0 & 1 & 1 & 4 \\ 0 & 3 & 1 & 6 \end{array}\right)$$

От третьей строки отнимаем вторую, умноженную на 3:

$$\tilde{A} \sim\left(\begin{array}{rrr|r} 1 & -1 & 0 & -2 \\ 0 & 1 & 1 & 4 \\ 0 & 0 & -2 & -6 \end{array}\right)$$

Умножив третью строку на $\left(-\frac{1}{2}\right)$ , получаем:

$$\tilde{A} \sim\left(\begin{array}{rrr|r} 1 & -1 & 0 & -2 \\ 0 & 1 & 1 & 4 \\ 0 & 0 & 1 & 3 \end{array}\right)$$

Проведем теперь обратный ход метода Гаусса (метод Гассу-Жордана), то есть сделаем нули над главной диагональю. Начнем с элементов третьего столбца. Надо обнулить элемент $a_{23}$, для этого от второй строки отнимем третью:

$$\tilde{A} \sim\left(\begin{array}{rrr|r} 1 & -1 & 0 & -2 \\ 0 & 1 & 0 & 1 \\ 0 & 0 & 1 & 3 \end{array}\right)$$

Далее обнуляем недиагональные элементы второго столбца, к первой строке прибавляем вторую:

$$\tilde{A} \sim\left(\begin{array}{ccc|c} 1 & 0 & 0 & -1 \\ 0 & 1 & 0 & 1 \\ 0 & 0 & 1 & 3 \end{array}\right)$$

Полученной матрице соответствует система

$\left\{\begin{array}{l}x_{1}+0 \cdot x_{2}+0 \cdot x_{3}=-1 \\ 0 \cdot x_{1}+x_{2}+0 \cdot x_{3}=1 \\ 0 \cdot x_{1}+0 \cdot x_{2}+x_{3}=3\end{array}\right.$    или   $\left\{\begin{array}{l} x_{1}=-1 \\ x_{2}=1 \\ x_{3}=3 \end{array}\right.$

Ответ. $\left\{\begin{array}{l} x_{1}=-1 \\ x_{2}=1 \\ x_{3}=3 \end{array}\right.$

Решение СЛАУ 3-его порядка методом Гаусса, пример № 11

СЛАУ 3-его порядка: 1 — 2 — 3 — 4 — 5 — 6 — 7 — 8 — 9 — 10 — 11 — 12
СЛАУ 4-ого порядка: 1 — 2 — 3 — 4 — 5 — 6 — 7 — 8 — 9 — 10 — 11 — 12


Условие

 5x 1 — x 2 — x 3   =   0
 x 1 + 2x 2 + 3x 3   =   14
 4x 1 + 3x 2 + 2x 3   =   16

Решение системы линейных алгебраических уравнений методом Гаусс

Для проверки ответов можете воспользоваться нашим онлайн сервисом — Решение системы линейных уравнений методом Гаусса. Все действия описанные в данном разделе не противоречат правилам обращения с матрицами и являются элементарными преобразованиями матрицы. Если после изучения примеров решения задач у Вас останутся вопросы, то Вы всегда можете задать их на форуме, и не забывайте про наши онлайн калькуляторы для решения задач по геометрии и другим предметам!

Перепишем систему линейных алгебраических уравнений в матричную форму. Получится матрица 3 × 4, слева от разделительной линии стоят коэффициенты при переменных, а справа стоят свободные члены.


Проведём следующие действия:

  • Поменяем местами строку № 1 и строку № 2

Получим:

Проведём следующие действия:

  • Из строки № 2 вычтем строку № 1 умноженную на 5 (Строка 2 — 5 × строка 1)
  • Из строки № 3 вычтем строку № 1 умноженную на 4 (Строка 3 — 4 × строка 1)

Получим:

Проведём следующие действия:

  • Строку № 3 поделим на -5 (Строка 3 = строка 3 / -5)
  • Поменяем местами строку № 2 и строку № 3

Получим:

Проведём следующие действия:

  • К строке № 3 прибавим строку № 2 умноженную на 11 (Строка 3 + 11 × строка 2)

Получим:

Проведём следующие действия:

  • Строку № 3 поделим на 6 (Строка 3 = строка 3 / 6)

Получим:

Проведём следующие действия:

  • Из строки № 2 вычтем строку № 3 умноженную на 2 (Строка 2 — 2 × строка 3)
  • Из строки № 1 вычтем строку № 3 умноженную на 3 (Строка 1 — 3 × строка 3)

Получим:

Проведём следующие действия:

  • Из строки № 1 вычтем строку № 2 умноженную на 2 (Строка 1 — 2 × строка 2)

Получим:

В левой части матрицы по главной диагонали остались одни единицы. В правом столбце получаем решение:
х1 = 1
х

2 = 2
х3 = 3


Вы поняли, как решать? Нет?

Помощь с решением

Метода Гаусса: примеры решения СЛАУ: понятия, определения, примеры задач

Найти решение этого же примера методом Гаусса в матричной форме записи:

3×1+2×2+x3+x4=-2×1-x2+4×3-x4=-1-2×1-2×2-3×3+x4=9×1+5×2-x3+2×4=4

Как решать?

Расширенная матрица системы представлена в виде:

   x1    x2     x3 x432111-14-1-2-2-3115-12-2-194

Прямой ход метода Гаусса в данном случае предполагает приведение расширенной матрицы к трапецеидальному виду при помощи элементарных преобразований. Этот процесс очень поход на процесс исключения неизвестных переменных в координатном виде.

Преобразование матрицы начинается с превращения всех элементов нулевые. Для этого к элементам 2-ой, 3-ей и 4-ой строк прибавляем соответствующие элементы 1-ой строки, которые умножены на -a21a11=-13, -a31a11=—23=23 и на -а41а11=-13.

Дальнейшие преобразования происходит по такой схеме: все элементы во 2-ом столбце, начиная с 3-ей строки, становятся нулевыми. Такой процесс соответствует процессу исключения переменной  . Для того, чтобы выполнить этой действие, необходимо к элементам 3-ей и 4-ой строк прибавить соответствующие элементы 1-ой строки матрицы, которая умножена на -а32(1)а22(1)=-23-53=-25 и -а42(1)а22(1)=-133-53=135:

   x1    x2     x3 x43211|-20-53113-43|-130-23-7353|2330133-4353|143~

      x1                 x2                           x3                           x4~3211|-20-53113-43|-130-23+(-25)(-53)-73+(-25)11353+(-25)(-43)|233+(-25)(-13)0133+135(-53)-43+135×11353+135(-43)|143+135(-13)~

       x1    x2     x3       x4~3211|-20-53113-43|-1300-195115|39500415-95|195

Теперь исключаем переменную x3 из последнего уравнения — прибавляем к элементам последней строки матрицы соответствующие элементы последней строки, которая умножена на а43(2)а33(2)=-415-195=4119.

       x1    x2     x3       x43211|-20-53113-43|-1300-195115|39500415-95|195~

      x1    x2               x3                           x4~3211|-20-53113-43|-1300-195115|39500415+4119(-195)-95+4119×115|195+4119×395~

       x1    x2     x3       x4~3211|-20-53113-43|-1300-195115|3950005619|39219

Теперь применим обратных ход метода. В матричной форме записи такое преобразование матрицы, чтобы матрица, которая отмечена цветом на изображении:

   x1    x2     x3       x43211|-20-53113-43|-1300-195115|3950005619|39219

стала диагональной, т.е. приняла следующий вид:

   x1    x2     x3       x43000|а10-5300|а200-1950|а30005619|39219, где а1, а2, а3 — некоторые числа.

Такие преобразования выступают аналогом прямому ходу, только преобразования выполняются не от 1-ой строки уравнения, а от последней. Прибавляем к элементам 3-ей, 2-ой и 1-ой строк соответствующие элементы последней строки, которая умножена на

-1155619=-209280, на —435619=1942 и на -15619=1956.

   x1    x2     x3       x43211|-20-53113-43|-1300-195115|3950005619|39219~

      x1    x2      x3                   x4~3211+(-1956)5619|-2+(-1956)392190-53113-43+1942×5619|-13+1942×3921900-195115+(-209280)5619|395+(-209280)392190005619|39219~

       x1    x2     x3       x4~3210|-90-531130|900-1950|-3850005619|39219

Далее прибавляем к элементам 2-ой и 1-ой строк соответствующие элементы 3-ей строки, которые умножены на

-113-195=5557 и на -1-195=519.

 x1    x2     x3       x43210|-90-531130|900-1950|-3850005619|39219~

      x1    x2             x3                   x4~321+519(-195)0|-9+519(-385)0-53113+5557(-195)0|9+5557(-385)00-1950|-3850005619|39219~

       x1    x2     x3       x4~3210|-110-5300|5300-1950|-3850005619|39219

На последнем этапе прибавляем элементы 2-ой строки к соответствующим элементам 1-ой строки, которые умножены на -2-53=65.

 x1    x2     x3       x43210|-110-5300|5300-1950|-3850005619|39219~

      x1           x2            x3      x4~32+65(-53)00|-11+65×53)0-5300|5300-1950|-3850005619|39219~

       x1    x2     x3       x4~3000|-90-5300|5300-1950|-3850005619|39219

Полученная матрица соответствует системе уравнений

3×1=-9-53×2=53-195×3=-3855619×4=39219, откуда находим неизвестные переменные.

Ответ: x1=-3, x2=-1,x3=2,x4=7.​​​

Метод Гаусса. Примеры

Метод Гаусса заключается в последовательном исключении переменных и преобразовании системы линейных алгебраических уравнений

к треугольному виду

Предположим, что в системе коэффициент . Если это условие не выполняется, то на первое место переносим уравнение, которое ее удовлетворяет. С помощью первого уравнения исключим из остальных уравнений.

Для этого делят первую строчку на , обозначим

.

Дальше второй строки вычитаем первую строку, умноженную на ;от третьего первую строчку, умноженный на ; и так далее до последней строки. Получим таблицу коэффициентов:

Для неизвестных имеем систему уравнений. Выполняя, как и раньше, исключим из всех уравнений, начиная с третьего. Для этого сначала разделим вторую строчку на .

Если коэффициент , то переставим уравнения так, чтобы выполнялось условие .

Обозначив

,

от третьей строки вычтем вторую строчку, умноженный на ;

от четвертой строки вычтем вторую строчку, умноженный на и т.д. Получим таблицу коэффициентов:

Продолжая процесс исключения неизвестных получим таблицу:

Таблица коэффициентов при неизвестных сводится к треугольному виду. Все главной диагонали элементы . Запишем соответствующую систему уравнений:

Переход от первой системы уравнений до последней называется прямым ходом метода Гаусса. Обратный ход метода Гаусса начинается с последней системы уравнений. Ее решают с конца до начала. Из последнего уравнения находят . Подставив это значение в предпоследнее — находят и т.д. Из первого уравнения находят .

Если система уравнений с неизвестными имеет единственное решение, то эта система всегда может быть преобразована к треугольному виду. Для студентов не всегда требуют, чтобы диагональные элементы были равны единице. Достаточно просто свести систему линейных уравнений к верхней треугольной.

———————————————

Пример 1.

Дана система трех линейных уравнений с тремя неизвестными. Решить систему методом Гаусса.

Решение.

Исключим неизвестную из второго и третьего уравнения. Для этого от них вычтем первое умноженное на

Видим, что наше уравнение в таком виде можно решать обратным ходом метода Гаусса. Для этого из последнего уравнения выразим

Подставим полученное значение в предыдущее уравнение и найдем

Из первого уравнения находим

Решение данной системы равен

——————————————

В случаях систем больших размеров, а также для удобства, часто на практике используют другую схему решения. Вместо преобразований над системой выполняют соответствующие преобразования над матрицей, составленной из коэффициентов при неизвестных и столбца из свободных членов, который для удобства выделяют вертикальной линией. Такую матрицу называют расширенной матрицей системы.

——————————————

Пример 2.

Решить систему четырех линейных алгебраических уравнений методом Гаусса.

Решение.

Выпишем расширенную матрицу для данной системы

Сведем ее к треугольному виду с помощью элементарных преобразований.

1.Поменяем местами первый и второй строки.

2. Добавим к элементам второго, третьего и четвертого строк элементы первой строки, умноженные соответственно на

3. Поменяем местами второй и третий строки. Добавим к элементам третьего и четвертого строк элементы второй строки, умноженные соответственно на

4. От четвертого уравнения умноженного на вычитаем третье уравнение умноженное на

Такой расширенной матрицы соответствует следующая система уравнений

С четвертого уравнения находим и подставляем в третье уравнение

Найденные значения подставляем во второе уравнение

Из первого уравнения находим первую неизвестную

Система полностью решена и – ее решение.

——————————————————

Посмотреть материалы:

в чем суть, решение системы уравнений, примеры с объяснением

Благодаря великим ученым было открыто множество эффективных теорем для работы со сложными математическими задачами. Один из таких примеров — метод Гаусса.

Метод Гаусса — что это такое

Метод Гаусса представляет собой методику эквивалентного преобразования исходной системы линейных уравнений в систему, решаемую существенно проще, чем исходный вариант.

Метод Гаусса используют для решения систем линейных алгебраических формул. Такой способ обладает рядом важных преимуществ:

  1. Нет необходимости сравнивать уравнения для оценки совместимости.
  2. Решение систем равенств, в которых число определителей совпадает или не совпадает с количеством неизвестных переменных.
  3. Поиск решений для уравнений с нулевым определителем.
  4. Сравнительно небольшое количество вычислительных операций для получения результата.

Основные определения и обозначения

Матрицы: определение и свойства

Такие системы являются наиболее удобным способом представления данных, с которыми впоследствии производят манипуляции. Матрица имеет вид прямоугольника для удобства расчетов. При использовании метода Гаусса работа осуществляется с треугольными матрицами, при записи которых применяется прямоугольник с нулями на тех местах, где числа отсутствуют. Часто нули не записывают, а только подразумевают.

Важным параметром матрицы является размер:

  • ширина — это количество строк, обозначают буквой m;
  • длину выражают числом столбцов, записывают буквой n.
Источник: bigpicture.ru

Размер матрицы будет записан в формате А m*n. В случае, когда m=n, матрица является квадратной, а m=n служит ее порядком. Номера строк и столбцов изменяются.

Определитель

Матрица обладает крайне важной характеристикой. Таким параметром является определитель. Данную величину рассчитывают с помощью диагонали. Для этого в матрице необходимо провести воображаемые диагональные линии. Затем следует найти произведение элементов, которые располагаются на этих диагоналях, а полученные значения суммировать таким образом:

  1. Если диагональ обладает наклоном в правую сторону, то знак «+».
  2. Для диагоналей, наклоненных влево, знак «–».
Источник: wp.com

Рассчитать определитель представляется возможным лишь в случае работы с квадратной матрицей.

Если необходимо определить данный параметр для прямоугольной матрицы, то следует выполнить следующие манипуляции:

  • из числа строк и числа столбцов выбрать наименьшее и обозначить его k;
  • отметить в матрице произвольным образом k столбцов и k строк.

Элементы, которые расположены на пересечении отмеченных столбцов и строк, образуют новую квадратную матрицу. В случае, когда определитель является числом, не равным нулю, то данный параметр будет обозначен как базисный минор первоначальной прямоугольной матрицы. Перед решением систем уравнений методом Гаусса полезно рассчитать определитель. Если данная характеристика равна нулю, то матрица имеет бесконечное множество решений либо не имеет их вовсе. В таком случае потребуется определить ранг матрицы.

Классификация систем

Ранг матрицы является распространенным понятием. Он обозначает максимальный порядок ее определителя, который не равен нулю. По-другому можно сказать, что ранг матрицы представляет собой порядок базисного минора. Исходя из данного критерия, СЛАУ классифицируют на несколько типов. В совместных системах, которые состоят лишь из коэффициентов, ранг основной матрицы совпадает с рангом расширенной. Для подобных систем характерно одно или множество решений. По этой причине совместные системы подразделяют на следующие типы:

  • определенные, обладающие одним решением, в которых наблюдается равенство ранга матрицы и количество неизвестных;
  • неопределенные;
  • обладающие бесконечным числом решений с рангом матрицы, который меньше количества неизвестных.

В несовместных системах ранги, характеризующие основную и расширенную матрицы, отличаются. С помощью метода Гаусса в процессе решения можно прийти либо к однозначному доказательству несовместности системы, либо к решению общего вида для системы, обладающей бесконечным количеством решений.

Источник: asiaplustj.info

Основные правила и разрешаемые преобразования при использовании метода Гаусса

Перед тем, как решать систему, необходимо ее упростить. На данном этапе выполняют элементарные преобразования, которые не влияют на конечный результат. Определенные манипуляции справедливы лишь в случае матриц, исходниками которых являются СЛАУ. Список элементарных преобразований:

  1. Перестановка строк. При перемене записей в системе местами ее решение не меняется. Можно менять место строк в матрице, учитывая столбец со свободными членами.
  2. Произведение всех элементов строк и некоторого коэффициента. Сокращаются большие числа в матрице, и исключаются нули. При этом множество решений сохраняется без изменений, а дальнейшие манипуляции существенно упрощаются. Важным условием является отличие от нуля коэффициента.
  3. Удаление строк, которые содержат пропорциональные коэффициенты. Данное преобразование следует из предыдущего пункта. При условии, что две или более строк в матрице обладают пропорциональными коэффициентами, то при произведении или делении одной из строк на коэффициент пропорциональности получают две или более абсолютно одинаковые строки. В этом случае лишние строки исключают, оставляя только одну.
  4. Удаление нулевой строки. Бывают случаи, когда в процессе манипуляций с уравнениями возникает строка, все элементы которой, в том числе свободный член, равны нулю. Нулевую строку допустимо исключать из матрицы.
  5. Суммирование элементов одной строки с элементами другой, умноженными на некоторый коэффициент, в соответствующих столбцах. Данное преобразование имеет наиболее важное значение из всех перечисленных.

Особенности использования метода Гаусса для решения СЛАУ

На первом этапе система уравнений записывается в определенном виде. Пример выглядит следующим образом:

Источник: wp.com

Коэффициенты необходимо представить в виде таблицы. С правой стороны в отдельном столбце записаны свободные члены. Данный блок отделен для удобства решения. Матрицу со столбцом со свободными членами называют расширенной.

Источник: wp.com

Затем основная матрица с коэффициентами приводится к верхней треугольной форме. Данное действие является ключевым моментом при решении системы уравнений с помощью метода Гаусса. По итогам преобразований матрица должна приобрести такой вид, чтобы слева внизу находились одни нули:

Источник: wp.com

При записи новой матрицы в виде системы уравнений можно отметить, что последняя строка уже содержит значение одного из корней, которое в дальнейшем подставляется в уравнение выше для нахождения следующего корня и так далее. Подобное описание позволяет разобраться в методе Гаусса в общих чертах.

Обратный и прямой ход метода Гаусса

В первом случае необходимо представить запись расширенной матрицы системы. При выполнении обратного метода Гаусса далее в главную матрицу добавляют столбец со свободными членами.

Источник: wp.com

Суть такого способа заключается в выполнении элементарных преобразований, по итогам которых данная матрица приводится к ступенчатому или треугольному виду. В этом случае над или под главной диагональю матрицы располагаются только нули.

Источник: wp.com

Варианты дальнейших действий:

  • перемена строк матрицы местами, при наличии одинаковых или пропорциональных строк их можно исключить, кроме одной;
  • деление либо умножение строки на любое число, не равное нулю;
  • удаление нулевых строк;
  • добавление строки, умноженной на число, не равное нулю, к другой строке.

Имея преобразованную систему с одной неизвестной Xn, которая становится известной, можно выполнить поиск в обратном порядке остальных неизвестных с помощью подстановки известных х в уравнения системы, вплоть до первого. Данный способ называют обратным методом Гаусса.

Примеры решений с объяснением

Пример 1

Требуется решить с помощью метода Гаусса систему линейных уравнений, которая выглядит следующим образом:

Источник: wp.com

Решение

Необходимо записать расширенную матрицу:

Источник: wp.com

Затем нужно выполнить преобразования. В результате матрица должна приобрести треугольный вид. Для этого следует умножить первую строку на (3) и умножить вторую строку на (-1). В результате суммирования второй и первой строк получается следующее:

Источник: wp.com

Далее следует умножить третью строку на (-1). После добавления третьей строки ко второй получаем следующие преобразования:

Источник: wp.com

После этого необходимо умножить первую строку на (6) и вторую строку на (13). Далее следует добавить вторую строку к первой:

Источник: wp.com

После того, как система преобразована, остается вычислить неизвестные:

\(x_{3}=\frac{98}{49}=2\)

\(x_{2}=\frac{14-7x_{3}}{6}=\frac{14-7*2}{6}=0\)

\(x_{3}=\frac{-9+5x_{2}+6x_{3}}{3}=\frac{-9+5*0+6*2}{3}=1\)

Данный пример демонстрирует единственное решение системы.

Источник: supertics.com

Пример 2

Необходимо решить систему уравнений, которая выглядит следующим образом:

Источник: wp.com

Решение

Необходимо составить матрицу:

Источник: wp.com

Согласно методу Гаусса уравнение первой строки по итогам преобразований не меняется. Удобнее, когда левый верхний элемент матрицы обладает наименьшим значением. В таком случае первые элементы остальных строк после преобразований будут равны нулю. Таким образом, составленная матрица будет решаться проще, если на место первой строки поставить вторую:

вторая строка:

\(k = (-a_{21} /a_{11}) = (-3/1) = -3\)

\(a»_{21} = a_{21} + k×a_{11} = 3 + (-3)×1 = 0\)

\(a» _{22} = a_{22} + k×a _{12} = -1 + (-3)×2 = -7\)

\(a»_{ 23} = a_{23} + k×a_{13} = 1 + (-3)×4 = -11\)

b» 2 = b 2 + k×b 1 = 12 + (-3)×12 = -24

третья строка: 

\(k = (-a_{31} /a_{11}) = (-5/1) = -5\)

\(a»_{31} = a_{31} + k×a_{11} = 5 + (-5)×1 = 0\)

\(a»_{32} = a_{32} + k×a_{12} = 1 + (-5)×2 = -9\)

\( a»_{33} = a_{33} + k×a_{13} = 2 + (-5)×4 = -18\)

\( b»_3 = b_3 + k×b_1 = 3 + (-5)×12 = -57\)

Матрица с промежуточными результатами манипуляций будет иметь следующий вид:

Источник: wp.com

Благодаря некоторым операциям можно придать матрице наиболее удобный вид. К примеру, вторую строку можно избавить от всех «минусов» путем умножения каждого элемента на «-1». Можно заметить, что для третьей строки характерны все элементы, кратные трем. В этом случае строка сокращается с помощью произведения каждого элемента на «-1/3». Минус позволит удалить отрицательные значения.

Источник: wp.com

Далее следует приступить к манипуляциям со второй и третьей строками. Необходимо суммировать третью и вторую строки. Вторая строка при этом умножается на такой коэффициент, при котором элемент а 32 будет равен нулю.

\(k = (-a_{32} /a_{22}) = (-3/7) = -3/7\)

В случае, когда некоторые преобразования приводят в результате к получению не целого числа, следует оставить его в этом виде. Таким образом, вычисления будут более точными. Затем при получении ответов можно определиться с его дальнейшем округлением или переводом в другую форму записи.

\(a»_{32} = a_{32} + k×a_{22} = 3 + (-3/7)×7 = 3 + (-3) = 0\)

\(a»_{33} = a_{33} + k×a_{23} = 6 + (-3/7)×11 = -9/7\)

\(b»_3 = b_3 + k×b_2 = 19 + (-3/7)×24 = -61/7\)

Преобразованная матрица будет иметь следующий вид:

 

 

Матрица обладает ступенчатым видом. Дальнейшие преобразования с помощью метода Гаусса нецелесообразны. В этом случае можно удалить из третьей строки общий коэффициент «-1/7».

Источник: wp.com

Затем необходимо представить запись матрицы в виде системы уравнений для вычисления корней.

x + 2y + 4z = 12 (1)

7y + 11z = 24 (2)

Найти корни можно обратным методом Гаусса. Уравнение (3) содержит значение z:

y = (24 — 11×(61/9))/7 = -65/9

С помощью первого уравнения можно определить х:

x = (12 — 4z — 2y)/1 = 12 — 4×(61/9) — 2×(-65/9) = -6/9 = -2/3

Подобная система является совместной и определенной, для которого характерно единственное решение. Ответ будет следующим:

x 1 = -2/3, y = -65/9, z = 61/9.

Метод Гаусса предполагает последовательное исключение неизвестных. Методика справедлива в случае решения квадратных систем линейных алгебраических уравнений. Несмотря на простоту метода, многие студенты сталкиваются с некоторыми трудностями в процессе поиска правильного решения. Это связано с наличием знаков «+» и «-». Поэтому для решения СЛАУ требуется проявить внимательность. А получить квалифицированную помощь можно на ресурсе Феникс.Хелп.

Системы линейных уравнений. Метод Гаусса

Рассмотрим систему линейных уравнений:

   

С этой системой связываются две матрицы: матрица коэффициентов

   

и расширенная матрица — с присоединенными свободными членами:

   

Элементарными преобразованиями системы линейных уравнений называются:

1. умножение уравнения на отличное от нуля число;

2. прибавление к одному уравнению любого другого, умноженного на любое число;

3. перестановка уравнений местами.

Теорема. Любая система линейных уравнений с помощью элементарных преобразований и, может быть, изменением нумерации неизвестных, может быть приведена к системе с трапециевидной матрицей.

Доказательство. Проводим элементарные преобразования только над строками матрицы , как в доказательстве теоремы о ранге матрицы. Возможно, при этом придется изменить нумерацию неизвестных. Приводим систему уравнений к виду

   

Если хотя бы одно из чисел отлично от нуля, то данная система уравнений решений не имеет (несовместна). Если же все они равны нулю, то последние равенств не несут никакой информации и могут быть отброшены. Тогда, если , то неизвестным можно придавать произвольные значения, а неизвестные находим из решения системы с треугольной матрицей

   

Эту систему удобно решать, определив из -го уравнения , затем из -го и т.д. Таким образом, можно выразить переменные через и получить общее решение системы. Если , то система (в случае совместности) имеет единственное решение.

Преобразование системы уравнений к системе с трапециевидной матрицей называется прямым ходом метода Гаусса. Последовательное вычисление неизвестных в порядке называется обратным ходом.

Пример. Решить систему линейных уравнений

   

Решение. Составим расширенную матрицу системы:

   

Первую строку умножим на 3 и вычтем из второй. Затем первую строку умножим на 2 и вычтем из третьей. Получим

   

Далее вторую строку прибавим к третьей и отбросим нулевую строку, получим

   

Запишем полученные уравнения:

   

Из второго уравнения выразим :

   

Полученное выражение подставляем в первое уравнение и выражаем из него :

   

Ответ. Общее решение данной системы:

   

Задачи.

1. Решите систему линейных уравнений

   

2. Решите систему линейных уравнений

   

3. Решите систему линейных уравнений

   

Решение систем линейных уравнений методом Гаусса

Содержание:

Решение систем линейных уравнений методом Гаусса

  • Решение системы линейных уравнений Метод Гаусса (1) Одним из наиболее универсальных и эффективных методов решения систем линейной алгебры является метод Гаусса. i + ai2X2 H —— h alnxn = bi a22 x2 t ‘• * T a2n xn- » am2®2 + •• + ​​GmUn = bn • Где (t, j = 2, m) — новое значение коэффициента, Правильная часть получена после первого шага. Аналогичным образом исключают неизвестные X2 из всех уравнений системы, учитывая основной элемент <4UФ0, исключая первое и второе.

    Примеры решения и задачи с методическими указаниями

    Решение задачЛекции
    Сборник и задачникУчебник
    • Продолжайте этот процесс как можно больше. Если процесс приведения системы (1) к постепенной форме показывает нулевые уравнения, то есть уравнения вида 0 = 0, они отбрасываются. Если отображается уравнение вида 0 = aΦ0>, это указывает на несовместимость системы. Второй шаг (обратный) — это решение ступенчатой ​​системы. В общем, существует множество решений системы градуированных уравнений. В последнем уравнении этой системы первое неизвестное xb представлено оставшимися неизвестными (£ fc + 1, …, xn). Затем подставьте значение Xk в предпоследнее уравнение системы и выразите Xk- \ через a: n).

    Тогда найди Xk-2> … Примечания: 1. Если система ступеней представляет собой треугольник, то есть k = 7i, исходная система имеет единственное решение. Найти xn из последнего уравнения и из второго уравнения xn-1) из последнего далее в систему всех остальных неизвестных [xn — 2? ••• yXi). 2.

    Прибавьте произвольные значения к свободным неизвестным …, xn), получите бесконечное число решений для системы. Людмила Фирмаль

    На практике удобнее выполнять все базовые преобразования для строк, используя матрицу расширения, а не систему (1). Удобно, если коэффициент aj равен 1 (переместить уравнение на место или отделить обе стороны уравнения все ф 1). Пример: 1) Решить систему, используя метод Гаусса. 2x \ -x-2 + 3×3-5 # 4 = 1, X \ -X2-bx3 = 2 3xi-2×2-2hz-5×4 = 3, 7xi-5×2-9hz-10×4 = 8.

    ♦ В результате базового преобразования в расширенную матрицу системы / 2-1 3 «-5 1 \ 1-1-5 0 2 3 -2 -2 -5 3 \ 7-5-9-10 8 / 1 -1 -5 0 2 \ 0 1 13 -5 -3 0 1 13 -5 -3 х0 2 26-10-6 / ^ 1 -1 -5 0 2 \ 2-13 -5 1 3 -2 -2 -5 3 ^ 7-5-9-10 8J -1 О 1 Ах ах \ 0 O -5 0 ‘2 л 13-5-3 LLC O O O y Оригинальная система была уменьшена до ступенчатой системы. xi-x2-5xs = 2 x2 + 13 Гц + 5×4 = -3. Итак, общее решение системы: x2 = -5×4-13x-X \ = -5×4-8×3-1. 1, x2 = x3 = 0, x4 = 0. 2) Решить систему, используя метод Гаусса. — = О, -3, ♦ X1 + x2 + x3 = 3, 2xi + 3×2 + 3×3 = 7, 3X] + X2 + x3 = 5, 5xi-x2-. Xs = 3. ♦ Выполнять базовые преобразования в строках расширенной матрицы системы.

    / 11 1 3 \ / 11 1 3 \ / 1 1 1 3 \ / 1 1 1 3 \ 2337 010 1 0101 0101 31 15 ~ 0-2—2-4 ~ 0112 ~ 0011 \ 5 -1 -1 3 / \ 0 -b -6 -12 / \ 0 I 1 2 / \ 0 0 0 0 / Полученная матрица соответствует системе + X-2 + xs = 3, X-2 = 1 Xb = 1. Выполнение обратного хода приводит к £ 3 = 1, x2-1, Xj = 1.

    Систем линейных уравнений: исключение Гаусса

    Системы линейных уравнений:
    Решение методом исключения Гаусса
    (стр. 6 из 7)

    Разделы: Определения, Решение по графику, Подстановка, Исключение / добавление, исключение по Гауссу.


    Решение трех переменных, линейных систем с тремя уравнениями сложнее, по крайней мере, на начальном этапе, чем решение систем с двумя переменными, потому что требуемые вычисления более грязный.Вы должны быть очень аккуратными в своей работе, и вы должны планируйте использовать много бумаги для заметок. Метод решения этих систем является расширением метода сложения двух переменных, поэтому сделайте конечно ты знаешь это метод хорошо и может использовать его последовательно правильно.

    Хотя метод решения основан на добавлении / исключении, попытка выполнить фактическое добавление имеет тенденцию становится очень запутанным, поэтому существует систематизированный метод решения трех или более переменных системы.Этот метод называется «исключение Гаусса» (с уравнения заканчиваются тем, что называется «строковой формой»).

    Начнем с простого, и работаем над более сложными примерами.

    • Решите следующие проблемы система уравнений.
    • Достаточно легко увидеть как действовать в этом случае. Я просто подставлю обратно значение z -value из третьего уравнения во второе, решите результат для л , г. а затем подключите z и y в первое уравнение и решите результат для x .

        10 л 3 (3) = 11
        10 y 9 = 11
        10 y = 20
        y = 2

        5x + 4 (2) (3) = 0
        5 x + 8 3 = 0
        5 x + 5 = 0
        5 x = 5
        x = 1

      Тогда решение ( х , y , z ) = (1, 2, 3).

    Причина, по которой эта система была Легко решить, что система была «треугольной»; это относится к уравнениям, имеющим форму треугольника, из-за нижних уравнений содержащий только более поздние переменные.

    Дело в том, что в этом формат, система проста в решении. И гауссовское исключение — это метод, который мы будем использовать для преобразования систем в эту верхнетреугольную форму, используя операции со строками, которые мы изучили, когда применили метод сложения.

    • Решите следующие проблемы система уравнений с использованием исключения Гаусса.
    • Уравнение не решается для переменной, поэтому мне нужно будет выполнить умножение и сложение чтобы упростить эту систему. Чтобы отслеживать свою работу, напишу вниз на каждом шагу, когда я иду. Но я сделаю свои вычисления на бумаге для заметок. Вот как я это сделал:

      Первое, что нужно сделать состоит в том, чтобы избавиться от ведущих x -термов в два ряда.А пока я просто посмотрю, какие строки будут легко расчистить; Я могу поменять строки позже, чтобы перевести систему в «верхний треугольной «формы. Нет правила, которое гласит, что я должен использовать x — срок из первой строки, и в этом случае, думаю, будет проще используйте термин x из третьей строки, так как его коэффициент просто «1». Поэтому я умножу третью строку на 3, и добавьте его в первую строку.Я делаю вычисления на бумаге для заметок:

      … а потом записываю результатов:

      (Когда мы решали системы с двумя переменными, мы могли умножить строку, переписав систему в сторону, а затем добавить. Для этого нет места в система с тремя переменными, поэтому нам и нужна бумага для заметок.) ​​

      Предупреждение: поскольку я не на самом деле ничего не делаю с третьей строкой, я скопировал ее без изменений, в новую матрицу уравнений.Я б / у третий ряд, но я на самом деле не менял Это. Не путайте «использование» с «изменением».

      Чтобы получить меньшие числа для коэффициентов умножу первую строку пополам:

      Теперь умножу третий ряд на 5 и добавьте это ко второму строка. Работаю на бумаге для заметок:

      … а потом записываю результаты: Авторские права Элизабет Стапель 2003-2011 Все права защищены

      Я ничего не делал с первым рядом, поэтому я скопировал его без изменений. Я работал с третий ряд, но я работал только на вторая строка, поэтому вторая строка обновляется, а третья строка копируется более без изменений.

      Хорошо, теперь x — столбец удаляется, за исключением ведущего члена в третьей строке.Так что дальше Приходится работать над колонкой и .

      Предупреждение: Начиная с третьего уравнение имеет член x , Я больше не могу использовать его ни в одном из двух других уравнений (или я отменить мой прогресс). Я могу работать с уравнением, но не с Это.

      Если я добавлю в два раза больше первого строки во вторую строку, это даст мне ведущую 1 во втором ряду.Я не буду избавились от ведущего y -терм во втором ряду, но я его преобразовал (не вмешиваясь дробями) в более простую форму. (Вы должны сохранить обратите внимание на такого рода упрощения.) Сначала я делаю царапину работа:

      … а потом записываю результатов:

    Теперь могу использовать второй ряд, чтобы убрать и -семестр в первом ряду.Вторую строку умножу на 7 и добавить. Сначала я царапаю работа:

    … а потом записываю результатов:

    Я могу сказать что z сейчас, но для большей точности я разделю первую строку на 43. Затем я переставляю ряды, чтобы придать им верхнетреугольную форму:

    Теперь я могу начать процесс обратного решения:

    Тогда решение ( х , y , z ) = ( 2, 3, 1 ) .

    Примечание: нет ничего священного о шагах, которые я использовал при решении указанной выше системы; там ничего не было Особо о том, как я решил эту систему. Вы могли бы работать в другом упорядочивайте или упрощайте разные строки, и все равно получите правильный ответ. Эти системы достаточно сложны, поэтому вряд ли один правильный способ вычисления ответа. Так что не беспокойтесь о том, «как она знала, что делать дальше? », потому что здесь нет правила.я просто делал все, что пришло мне в голову; Я делал то, что казалось простейшим, или как пришла в голову первая. Не волнуйтесь, если бы вы использовали совершенно другой шаги. Если каждый шаг на пути верен, вы придумаете Такой же ответ.


    В приведенном выше примере я мог пошли дальше в своих вычислениях и более тщательно проработали строковые операции, очищая все термины и кроме этого во второй строке и во всех терминах z кроме того, что в первой строке.Это то, что процесс тогда выглядело так:

    Так я могу просто читать от значений x , л , г. и z , и мне не нужно возиться с обратной заменой. Это более полное метод решения называется «методом исключения Гаусса-Жордана» (с уравнения, попадающие в так называемый «пониженный ряд-эшелон» форма»).Многие тексты доходят до исключения Гаусса, но я всегда было легче продолжать и делать Гаусс-Джордан.

    Обратите внимание, что я выполнил две строковые операции сразу на этом последнем шаге перед переключением строк. Пока я не работая с и работая на в той же строке на том же шаге, это нормально. В этом случае я работал с первой строкой и рабочая по второй и третий ряды.

    << Предыдущая Вверх | 1 | 2 | 3 | 4 | 5 | 6 | 7 | Вернуться к указателю Далее >>

    Цитируйте эту статью как:

    Стапель, Елизавета. «Системы линейных уравнений, решаемые методом исключения Гаусса». Purplemath
    Доступно по телефону https: // www.purplemath.com/modules/systlin6.htm .
    Доступ [Дата] [Месяц] 2016 г.

    Решение системы с исключением Гаусса

    Результаты обучения

    • Используйте метод исключения Гаусса для решения системы уравнений, представленной в виде расширенной матрицы.
    • Интерпретировать решение системы уравнений, представленной в виде расширенной матрицы.

    Мы увидели, как написать систему уравнений с расширенной матрицей , а затем как использовать строковые операции и обратную подстановку, чтобы получить форму с эшелонированием строк .Теперь мы будем использовать метод исключения Гаусса как инструмент для решения системы, записанной в виде расширенной матрицы. В нашем первом примере мы покажем вам процесс использования исключения Гаусса в системе двух уравнений с двумя переменными.

    Пример: решение системы 2 X 2 методом исключения Гаусса

    Решите данную систему методом исключения Гаусса.

    [латекс] \ begin {array} {l} 2x + 3y = 6 \ hfill \\ \ text {} x-y = \ frac {1} {2} \ hfill \ end {array} [/ latex]

    Показать решение

    Сначала запишем это как расширенную матрицу.

    [латекс] \ left [\ begin {array} {rr} \ hfill 2 & \ hfill 3 \\ \ hfill 1 & \ hfill -1 \ end {array} \ text {} | \ text {} \ begin {array} { r} \ hfill 6 \\ \ hfill \ frac {1} {2} \ end {array} \ right] [/ latex]

    Нам нужна 1 в строке 1, столбце 1. Этого можно добиться, поменяв местами строку 1 и строку 2.

    [латекс] {R} _ {1} \ leftrightarrow {R} _ {2} \ to \ left [\ begin {array} {rrr} \ hfill 1 & \ hfill -1 & \ hfill \\ \ hfill 2 & \ hfill 3 & \ hfill \ end {array} | \ begin {array} {rr} \ hfill & \ hfill \ frac {1} {2} \\ \ hfill & \ hfill 6 \ end {array} \ right] [/ latex]

    Теперь у нас есть 1 как первая запись в строке 1, столбце 1.Теперь давайте получим 0 в строке 2, столбце 1. Это можно сделать, умножив строку 1 на [latex] -2 [/ latex], а затем прибавив результат к строке 2.

    [латекс] -2 {R} _ {1} + {R} _ {2} = {R} _ {2} \ to \ left [\ begin {array} {rrr} \ hfill 1 & \ hfill -1 & \ hfill \\ \ hfill 0 & \ hfill 5 & \ hfill \ end {array} | \ begin {array} {rr} \ hfill & \ hfill \ frac {1} {2} \\ \ hfill & \ hfill 5 \ end {массив } \ right] [/ latex]

    У нас есть только один шаг, чтобы умножить строку 2 на [latex] \ frac {1} {5} [/ latex].

    [латекс] \ frac {1} {5} {R} _ {2} = {R} _ {2} \ to \ left [\ begin {array} {rrr} \ hfill 1 & \ hfill -1 & \ hfill \ \ \ hfill 0 & \ hfill 1 & \ hfill \ end {array} | \ begin {array} {cc} & \ frac {1} {2} \\ & 1 \ end {array} \ right] [/ latex]

    Использовать обратную замену.Вторая строка матрицы представляет [латекс] y = 1 [/ латекс]. Подставьте обратно [latex] y = 1 [/ latex] в первое уравнение.

    [латекс] \ begin {array} {l} x- \ left (1 \ right) = \ frac {1} {2} \ hfill \\ \ text {} x = \ frac {3} {2} \ hfill \ end {array} [/ latex]

    Решение — точка [латекс] \ left (\ frac {3} {2}, 1 \ right) [/ latex].

    Попробуй

    Решите данную систему методом исключения Гаусса.

    [латекс] \ begin {массив} {l} 4x + 3y = 11 \ hfill \\ \ text {} \ text {} \ text {} x — 3y = -1 \ hfill \ end {array} [/ latex]

    Показать решение

    [латекс] \ влево (2,1 \ вправо) [/ латекс]

    В нашем следующем примере мы решим систему двух уравнений с двумя зависимыми переменными.Напомним, что зависимая система имеет бесконечное количество решений, и результатом операций со строками в ее расширенной матрице будет уравнение, такое как [latex] 0 = 0 [/ latex]. Мы также рассмотрим написание общего решения для зависимой системы.

    Пример: решение зависимой системы

    Решите систему уравнений.

    [латекс] \ begin {array} {l} 3x + 4y = 12 \\ 6x + 8y = 24 \ end {array} [/ latex]

    Показать решение Выполните строковые операции на расширенной матрице, чтобы попытаться получить строковую форму .

    [латекс] A = \ left [\ begin {array} {llll} 3 \ hfill & \ hfill & 4 \ hfill & \ hfill \\ 6 \ hfill & \ hfill & 8 \ hfill & \ hfill \ end {array} | \ begin {array} {ll} \ hfill & 12 \ hfill \\ \ hfill & 24 \ hfill \ end {array} \ right] [/ latex]

    [латекс] \ begin {array} {l} \ hfill \\ \ begin {array} {l} — \ frac {1} {2} {R} _ {2} + {R} _ {1} = { R} _ {1} \ to \ left [\ begin {array} {llll} 0 \ hfill & \ hfill & 0 \ hfill & \ hfill \\ 6 \ hfill & \ hfill & 8 \ hfill & \ hfill \ end { array} | \ begin {array} {ll} \ hfill & 0 \ hfill \\ \ hfill & 24 \ hfill \ end {array} \ right] \ hfill \\ {R} _ {1} \ leftrightarrow {R} _ {2} \ to \ left [\ begin {array} {llll} 6 \ hfill & \ hfill & 8 \ hfill & \ hfill \\ 0 \ hfill & \ hfill & 0 \ hfill & \ hfill \ end {array} | \ begin {array} {ll} \ hfill & 24 \ hfill \\ \ hfill & 0 \ hfill \ end {array} \ right] \ hfill \ end {array} \ hfill \ end {array} [/ latex]

    Матрица заканчивается всеми нулями в последней строке: [latex] 0y = 0 [/ latex].Таким образом, существует бесконечное количество решений и система классифицируется как зависимая. Чтобы найти общее решение, вернитесь к одному из исходных уравнений и решите для [latex] y [/ latex].

    [латекс] \ begin {array} {l} 3x + 4y = 12 \ hfill \\ \ text {} 4y = 12 — 3x \ hfill \\ \ text {} y = 3- \ frac {3} {4} x \ hfill \ end {array} [/ latex]

    Итак, решение этой системы — [латекс] \ left (x, 3- \ frac {3} {4} x \ right) [/ latex].

    Теперь мы перейдем на следующий шаг к решению системы линейных уравнений 3 на 3.Общая идея состоит в том, чтобы исключить все переменные, кроме одной, с помощью операций со строками, а затем выполнить обратную замену для поиска других переменных.

    Пример: решение системы линейных уравнений с использованием матриц

    Решите систему линейных уравнений с помощью матриц.

    [латекс] \ begin {массив} {c} \ begin {array} {l} \ hfill \\ \ hfill \\ x-y + z = 8 \ hfill \ end {array} \\ 2x + 3y-z = -2 \\ 3x — 2y — 9z = 9 \ end {array} [/ latex]

    Показать решение

    Сначала мы пишем расширенную матрицу.

    [латекс] \ left [\ begin {array} {rrr} \ hfill 1 & \ hfill -1 & \ hfill 1 \\ \ hfill 2 & \ hfill 3 & \ hfill -1 \\ \ hfill 3 & \ hfill -2 & \ hfill -9 \ end {array} \ text {} | \ text {} \ begin {array} {r} \ hfill 8 \\ \ hfill -2 \\ \ hfill 9 \ end {array} \ right] [/ latex]

    Затем мы выполняем операции со строками, чтобы получить форму строки-эшелона.

    [латекс] \ begin {array} {rrrrr} \ hfill -2 {R} _ {1} + {R} _ {2} = {R} _ {2} \ to \ left [\ begin {array} { rrrrrr} \ hfill 1 & \ hfill & \ hfill -1 & \ hfill & \ hfill 1 & \ hfill \\ \ hfill 0 & \ hfill & \ hfill 5 & \ hfill & \ hfill -3 & \ hfill \\ \ hfill 3 & \ hfill & \ hfill -2 & \ hfill & \ hfill -9 & \ hfill \ end {array} | \ begin {array} {rr} \ hfill & \ hfill 8 \\ \ hfill & \ hfill -18 \\ \ hfill & \ hfill 9 \ end {массив} \ right] & \ hfill & \ hfill & \ hfill & \ hfill -3 {R} _ {1} + {R} _ {3} = {R} _ {3} \ to \ left [\ begin {array} {rrrrrr} \ hfill 1 & \ hfill & \ hfill -1 & \ hfill & \ hfill 1 & \ hfill \\ \ hfill 0 & \ hfill & \ hfill 5 & \ hfill & \ hfill -3 & \ hfill \\ \ hfill 0 & \ hfill & \ hfill 1 & \ hfill & \ hfill -12 & \ hfill \ end {array} | \ begin {array} {rr} \ hfill & \ hfill 8 \\ \ hfill & \ hfill -18 \\ \ hfill & \ hfill -15 \ end {array} \ right] \ end {array} [/ latex]

    Самый простой способ получить 1 в строке 2 столбца 1 — это поменять местами [латекс] {R} _ {2} [/ latex] и [latex] {R} _ {3} [/ latex].

    [латекс] \ text {Interchange} {R} _ {2} \ text {и} {R} _ {3} \ to \ left [\ begin {array} {rrrrrrr} \ hfill 1 & \ hfill & \ hfill — 1 & \ hfill & \ hfill 1 & \ hfill & \ hfill 8 \\ \ hfill 0 & \ hfill & \ hfill 1 & \ hfill & \ hfill -12 & \ hfill & \ hfill -15 \\ \ hfill 0 & \ hfill & \ hfill 5 & \ hfill & \ hfill -3 & \ hfill & \ hfill -18 \ end {array} \ right] [/ latex]

    Затем

    [латекс] \ begin {array} {l} \\ \ begin {array} {rrrrr} \ hfill -5 {R} _ {2} + {R} _ {3} = {R} _ {3} \ в \ left [\ begin {array} {rrrrrr} \ hfill 1 & \ hfill & \ hfill -1 & \ hfill & \ hfill 1 & \ hfill \\ \ hfill 0 & \ hfill & \ hfill 1 & \ hfill & \ hfill -12 & \ hfill \\ \ hfill 0 & \ hfill & \ hfill 0 & \ hfill & \ hfill 57 & \ hfill \ end {array} | \ begin {array} {rr} \ hfill & \ hfill 8 \\ \ hfill & \ hfill -15 \\ \ hfill & \ hfill 57 \ end {array} \ right] & \ hfill & \ hfill & \ hfill & \ hfill — \ frac {1} {57} {R} _ {3} = {R} _ {3} \ to \ left [\ begin {array} {rrrrrr} \ hfill 1 & \ hfill & \ hfill -1 & \ hfill & \ hfill 1 & \ hfill \\ \ hfill 0 & \ hfill & \ hfill 1 & \ hfill & \ hfill -12 & \ hfill \\ \ hfill 0 & \ hfill & \ hfill 0 & \ hfill & \ hfill 1 & \ hfill \ end {array} | \ begin {array} {rr} \ hfill & \ hfill 8 \\ \ hfill & \ hfill -15 \ \ \ hfill & \ hfill 1 \ end {array} \ right] \ end {array} \ end {array} [/ latex]

    Последняя матрица представляет собой эквивалентную систему.

    [латекс] \ begin {массив} {l} \ text {} x-y + z = 8 \ hfill \\ \ text {} y — 12z = -15 \ hfill \\ \ text {} z = 1 \ hfill \ end {array} [/ latex]

    Используя обратную подстановку, мы получаем решение как [latex] \ left (4, -3,1 \ right) [/ latex].

    Напомним, что есть три возможных исхода решений для линейных систем. В предыдущем примере решение [латекс] \ left (4, -3,1 \ right) [/ latex] представляет точку в трехмерном пространстве. Эта точка представляет собой пересечение трех плоскостей.В следующем примере мы решаем систему, используя операции со строками, и обнаруживаем, что она представляет зависимую систему. Зависимая система в 3-х измерениях может быть представлена ​​двумя идентичными плоскостями, как в 2-х измерениях, где зависимая система представляет две идентичные линии.

    Пример: решение зависимой системы 3 x 3

    Решите следующую систему линейных уравнений, используя метод исключения Гаусса.

    [латекс] \ begin {array} {r} \ hfill -x — 2y + z = -1 \\ \ hfill 2x + 3y = 2 \\ \ hfill y — 2z = 0 \ end {array} [/ latex]

    Показать решение

    Запишите расширенную матрицу.

    [латекс] \ left [\ begin {array} {rrr} \ hfill -1 & \ hfill -2 & \ hfill 1 \\ \ hfill 2 & \ hfill 3 & \ hfill 0 \\ \ hfill 0 & \ hfill 1 & \ hfill -2 \ end {array} \ text {} | \ text {} \ begin {array} {r} \ hfill -1 \\ \ hfill 2 \\ \ hfill 0 \ end {array} \ right] [/ latex]

    Сначала умножьте строку 1 на [latex] -1 [/ latex], чтобы получить 1 в строке 1, столбце 1. Затем выполните операции со строками , чтобы получить форму строки-эшелон.

    [латекс] — {R} _ {1} \ to \ left [\ begin {array} {rrrrrrr} \ hfill 1 & \ hfill & \ hfill 2 & \ hfill & \ hfill -1 & \ hfill & \ hfill 1 \\ \ hfill 2 & \ hfill & \ hfill 3 & \ hfill & \ hfill 0 & \ hfill & \ hfill 2 \\ \ hfill 0 & \ hfill & \ hfill 1 & \ hfill & \ hfill -2 & \ hfill & \ hfill 0 \ end {array} \ справа] [/ латекс]

    [латекс] {R} _ {2} \ leftrightarrow {R} _ {3} \ to \ left [\ begin {array} {rrrrr} \ hfill 1 & \ hfill & \ hfill 2 & \ hfill & \ hfill -1 \ \ \ hfill 0 & \ hfill & \ hfill 1 & \ hfill & \ hfill -2 \\ \ hfill 2 & \ hfill & \ hfill 3 & \ hfill & \ hfill 0 \ end {array} \ text {} | \ begin {array} { rr} \ hfill & \ hfill 1 \\ \ hfill & \ hfill 0 \\ \ hfill & \ hfill 2 \ end {array} \ right] [/ latex]

    [латекс] -2 {R} _ {1} + {R} _ {3} = {R} _ {3} \ to \ left [\ begin {array} {rrrrrr} \ hfill 1 & \ hfill & \ hfill 2 & \ hfill & \ hfill -1 & \ hfill \\ \ hfill 0 & \ hfill & \ hfill 1 & \ hfill & \ hfill -2 & \ hfill \\ \ hfill 0 & \ hfill & \ hfill -1 & \ hfill & \ hfill 2 & \ hfill \ end {array} | \ begin {array} {rr} \ hfill & \ hfill 1 \\ \ hfill & \ hfill 0 \\ \ hfill & \ hfill 0 \ end {array} \ right] [/ latex]

    [латекс] {R} _ {2} + {R} _ {3} = {R} _ {3} \ to \ left [\ begin {array} {rrrrrr} \ hfill 1 & \ hfill & \ hfill 2 & \ hfill & \ hfill -1 & \ hfill \\ \ hfill 0 & \ hfill & \ hfill 1 & \ hfill & \ hfill -2 & \ hfill \\ \ hfill 0 & \ hfill & \ hfill 0 & \ hfill & \ hfill 0 & \ hfill \ end { array} | \ begin {array} {rr} \ hfill & \ hfill 2 \\ \ hfill & \ hfill 1 \\ \ hfill & \ hfill 0 \ end {array} \ right] [/ latex]

    Последняя матрица представляет следующую систему.

    [латекс] \ begin {массив} {l} \ text {} x + 2y-z = 1 \ hfill \\ \ text {} y — 2z = 0 \ hfill \\ \ text {} 0 = 0 \ hfill \ конец {array} [/ latex]

    По тождеству [latex] 0 = 0 [/ latex] мы видим, что это зависимая система с бесконечным числом решений. Затем мы находим общее решение. Решив второе уравнение для [latex] y [/ latex] и подставив его в первое уравнение, мы можем решить для [latex] z [/ latex] через [latex] x [/ latex].

    [латекс] \ begin {array} {l} \ text {} x + 2y-z = 1 \ hfill \\ \ text {} y = 2z \ hfill \\ \ hfill \\ x + 2 \ left (2z \ справа) -z = 1 \ hfill \\ \ text {} x + 3z = 1 \ hfill \\ \ text {} z = \ frac {1-x} {3} \ hfill \ end {array} [/ latex]

    Теперь мы подставляем выражение для [latex] z [/ latex] во второе уравнение, чтобы решить для [latex] y [/ latex] через [latex] x [/ latex].

    [латекс] \ begin {массив} {l} \ text {} y — 2z = 0 \ hfill \\ \ text {} z = \ frac {1-x} {3} \ hfill \\ \ hfill \\ y — 2 \ left (\ frac {1-x} {3} \ right) = 0 \ hfill \\ \ text {} y = \ frac {2 — 2x} {3} \ hfill \ end {array} [/ latex ]

    Общее решение — [latex] \ left (x, \ frac {2 — 2x} {3}, \ frac {1-x} {3} \ right) [/ latex].

    Общее решение для зависимой системы 3 X 3

    Напомним, что когда вы решаете зависимую систему линейных уравнений с двумя переменными с использованием исключения или подстановки, вы можете записать решение [latex] (x, y) [/ latex] через x, потому что существует бесконечно много (x, y) пары, которые будут удовлетворять зависимой системе уравнений, и все они попадают на линию [латекс] (x, mx + b) [/ latex].Теперь, когда вы работаете в трех измерениях, решение будет представлять собой плоскость, поэтому вы должны записать его в общей форме [латекс] (x, m_ {1} x + b_ {1}, m_ {2} x + b_ { 2}) [/ латекс].

    Попробуй

    Решите систему методом исключения Гаусса.

    [латекс] \ begin {array} {c} x + 4y-z = 4 \\ 2x + 5y + 8z = 15 \ x + 3y — 3z = 1 \ end {array} [/ latex]

    Показать решение

    [латекс] \ левый (1,1,1 \ правый) [/ латекс]

    Вопросы и ответы

    Можно ли решить любую систему линейных уравнений методом исключения Гаусса?

    Да, система линейных уравнений любого размера может быть решена методом исключения Гаусса.

    Как: решить систему уравнений с помощью матриц с помощью калькулятора

    1. Сохраните расширенную матрицу как матричную переменную [latex] \ left [A \ right], \ left [B \ right], \ left [C \ right] \ text {,} \ dots [/ latex].
    2. Используйте функцию ref ( в калькуляторе, вызывая каждую матричную переменную по мере необходимости.

    Пример: решение систем уравнений с помощью калькулятора

    Решите систему уравнений.

    [латекс] \ begin {array} {r} \ hfill 5x + 3y + 9z = -1 \\ \ hfill -2x + 3y-z = -2 \\ \ hfill -x — 4y + 5z = 1 \ end { array} [/ latex]

    Показать решение

    Напишите расширенную матрицу для системы уравнений.

    [латекс] \ left [\ begin {array} {rrr} \ hfill 5 & \ hfill 3 & \ hfill 9 \\ \ hfill -2 & \ hfill 3 & \ hfill -1 \\ \ hfill -1 & \ hfill -4 & \ hfill 5 \ end {array} \ text {} | \ text {} \ begin {array} {r} \ hfill -1 \\ \ hfill -2 \\ \ hfill 1 \ end {array} \ right] [/ latex]

    На странице матриц калькулятора введите расширенную матрицу выше как матричную переменную [latex] \ left [A \ right] [/ latex].

    [латекс] \ left [A \ right] = \ left [\ begin {array} {rrrrrrr} \ hfill 5 & \ hfill & \ hfill 3 & \ hfill & \ hfill 9 & \ hfill & \ hfill -1 \\ \ hfill — 2 & \ hfill & \ hfill 3 & \ hfill & \ hfill -1 & \ hfill & \ hfill -2 \\ \ hfill -1 & \ hfill & \ hfill -4 & \ hfill & \ hfill 5 & \ hfill & \ hfill 1 \ end {массив } \ right] [/ latex]

    Используйте функцию ref ( в калькуляторе, вызывая матричную переменную [latex] \ left [A \ right] [/ latex].

    [латекс] \ text {ref} \ left (\ left [A \ right] \ right) [/ латекс]

    Оценить.

    [латекс] \ begin {array} {l} \ hfill \\ \ left [\ begin {array} {rrrr} \ hfill 1 & \ hfill \ frac {3} {5} & \ hfill \ frac {9} {5 } & \ hfill \ frac {1} {5} \\ \ hfill 0 & \ hfill 1 & \ hfill \ frac {13} {21} & \ hfill — \ frac {4} {7} \\ \ hfill 0 & \ hfill 0 & \ hfill 1 & \ hfill — \ frac {24} {187} \ end {array} \ right] \ to \ begin {array} {l} x + \ frac {3} {5} y + \ frac {9} {5} z = — \ frac {1} {5} \ hfill \\ \ text {} y + \ frac {13} {21} z = — \ frac {4} {7} \ hfill \\ \ text {} z = — \ frac {24} {187} \ hfill \ end {array} \ hfill \ end {array} [/ latex]

    При использовании обратной подстановки решение: [latex] \ left (\ frac {61} {187}, — \ frac {92} {187}, — \ frac {24} {187} \ right) [/ latex] .

    Приложения систем уравнений

    Теперь обратимся к приложениям, для которых используются системы уравнений. В следующем примере мы определяем, сколько денег было инвестировано по двум разным ставкам, учитывая сумму процентов, полученных на обоих счетах.

    Пример: применение матриц 2 × 2 к финансам

    Кэролайн инвестирует в общей сложности 12 000 долларов в две муниципальные облигации, одна из которых выплачивает 10,5% годовых, а другая — 12%. Годовой процент, полученный по двум инвестициям в прошлом году, составил 1335 долларов.Сколько было вложено по каждой ставке?

    Показать решение

    У нас есть система двух уравнений с двумя переменными. Пусть [latex] x = [/ latex] сумма, инвестированная под 10,5% годовых, и [latex] y = [/ latex] сумма, инвестированная под 12% годовых.

    [латекс] \ begin {array} {l} \ text {} x + y = 12 000 \ hfill \\ 0.105x + 0.12y = 1335 \ hfill \ end {array} [/ latex]

    В качестве матрицы имеем

    [латекс] \ left [\ begin {array} {rr} \ hfill 1 & \ hfill 1 \\ \ hfill 0.105 & \ hfill 0.12 \ end {array} \ text {} | \ text {} \ begin {array} { r} \ hfill 12 000 \\ \ hfill 1,335 \ end {array} \ right] [/ latex]

    Умножить строку 1 на [латекс] -0.105 [/ latex] и добавьте результат в строку 2.

    [латекс] \ left [\ begin {array} {rr} \ hfill 1 & \ hfill 1 \\ \ hfill 0 & \ hfill 0.015 \ end {array} \ text {} | \ text {} \ begin {array} {r } \ hfill 12,000 \\ \ hfill 75 \ end {array} \ right] [/ latex]

    Затем,

    [латекс] \ begin {array} {l} 0,015y = 75 \ hfill \\ \ text {} y = 5,000 \ hfill \ end {array} [/ latex]

    Итак [латекс] 12 000 — 5 000 = 7 000 [/ латекс].

    Таким образом, 5000 долларов были инвестированы под 12% годовых и 7000 долларов под 10,5%.

    Пример: применение матриц 3 × 3 к финансам

    Ava инвестирует в общей сложности 10 000 долларов в три счета, один из которых платит 5% годовых, другой — 8%, а третий — 9%.Годовой процент, полученный по трем инвестициям в прошлом году, составил 770 долларов. Сумма, вложенная под 9%, была вдвое больше, чем сумма, вложенная под 5%. Сколько было вложено по каждой ставке?

    Показать решение

    У нас есть система трех уравнений с тремя переменными. Пусть [latex] x [/ latex] будет сумма, инвестированная под 5% годовых, пусть [latex] y [/ latex] будет суммой, инвестированной под 8%, и пусть [latex] z [/ latex] будет инвестированной суммой. под 9% годовых. Таким образом,

    [латекс] \ begin {array} {l} \ text {} x + y + z = 10 000 \ hfill \\ 0.05x + 0,08y + 0,09z = 770 \ hfill \\ \ text {} 2x-z = 0 \ hfill \ end {array} [/ latex]

    В качестве матрицы имеем

    [латекс] \ left [\ begin {array} {rrr} \ hfill 1 & \ hfill 1 & \ hfill 1 \\ \ hfill 0,05 & \ hfill 0,08 & \ hfill 0,09 \\ \ hfill 2 & \ hfill 0 & \ hfill -1 \ end {array} \ text {} | \ text {} \ begin {array} {r} \ hfill 10,000 \\ \ hfill 770 \\ \ hfill 0 \ end {array} \ right] [/ latex]

    Теперь мы выполняем исключение Гаусса, чтобы получить форму строки-эшелон.

    [латекс] \ begin {массив} {l} \ begin {array} {l} \ hfill \\ -0.05 {R} _ {1} + {R} _ {2} = {R} _ {2} \ to \ left [\ begin {array} {rrrrrr} \ hfill 1 & \ hfill & \ hfill 1 & \ hfill & \ hfill 1 & \ hfill \\ \ hfill 0 & \ hfill & \ hfill 0.03 & \ hfill & \ hfill 0.04 & \ hfill \\ \ hfill 2 & \ hfill & \ hfill 0 & \ hfill & \ hfill -1 & \ hfill \ end {array} | \ begin {array} {rr} \ hfill & \ hfill 10,000 \\ \ hfill & \ hfill 270 \\ \ hfill & \ hfill 0 \ end {array} \ right] \ hfill \ end {array} \ hfill \\ -2 {R} _ {1} + {R} _ {3} = {R} _ {3} \ to \ left [\ begin {array} {rrrrrr} \ hfill 1 & \ hfill & \ hfill 1 & \ hfill & \ hfill 1 & \ hfill \\ \ hfill 0 & \ hfill & \ hfill 0.03 & \ hfill & \ hfill 0.04 & \ hfill \\ \ hfill 0 & \ hfill & \ hfill -2 & \ hfill & \ hfill -3 & \ hfill \ end {array} | \ begin {array} {rr} \ hfill & \ hfill 10,000 \\ \ hfill & \ hfill 270 \\ \ hfill & \ hfill -20,000 \ end {array} \ right] \ hfill \\ \ frac {1} {0.03} {R} _ {2} = {R} _ {2} \ to \ left [\ begin {array} {rrrrrr} \ hfill 0 & \ hfill & \ hfill 1 & \ hfill & \ hfill 1 & \ hfill \\ \ hfill 0 & \ hfill & \ hfill 1 & \ hfill & \ hfill \ frac {4} {3} & \ hfill \\ \ hfill 0 & \ hfill & \ hfill -2 & \ hfill & \ hfill -3 & \ hfill \ end {array} | \ begin {array} {rr} \ hfill & \ hfill 10,000 \\ \ hfill & \ hfill 9,000 \\ \ hfill & \ hfill -20,000 \ end {array} \ right] \ hfill \\ 2 {R} _ {2} + {R} _ {3} = {R} _ {3} \ to \ left [\ begin {array} {rrrrrr} \ hfill 1 & \ hfill & \ hfill 1 & \ hfill & \ hfill 1 & \ hfill \\ \ hfill 0 & \ hfill & \ hfill 1 & \ hfill & \ hfill \ frac {4} {3} & \ hfill \\ \ hfill 0 & \ hfill & \ hfill 0 & \ hfill & \ hfill — \ frac {1} {3} & \ hfill \ end {array} | \ begin {array} {rr} \ hfill & \ hfill 10,000 \\ \ hfill & \ hfill 9,000 \\ \ hfill & \ hfill -2,000 \ end {array} \ right] \ hfill \ end {array} [/ latex]

    Третья строка сообщает нам [латекс] — \ frac {1} {3} z = -2,000 [/ latex]; таким образом [латекс] z = 6,000 [/ латекс].

    Вторая строка сообщает нам [латекс] y + \ frac {4} {3} z = 9000 [/ latex].

    Подставляя [латекс] z = 6,000 [/ латекс], получаем

    [латекс] \ begin {array} {r} \ hfill y + \ frac {4} {3} \ left (6000 \ right) = 9000 \\ \ hfill y + 8000 = 9000 \\ \ hfill y = 1000 \ end {array} [/ latex]

    Первая строка сообщает нам [латекс] x + y + z = 10,000 [/ latex]. Подставив [latex] y = 1,000 [/ latex] и [latex] z = 6,000 [/ latex], мы получим
    [latex] \ begin {array} {l} x + 1,000 + 6,000 = 10,000 \ hfill \\ \ text {} x = 3 000 \ text {} \ hfill \ end {array} [/ latex]

    Ответ: 3000 долларов вложены под 5%, 1000 долларов вложены под 8% и 6000 долларов вложены под 9%.

    Попробуй

    Небольшая обувная компания взяла ссуду в размере 1 500 000 долларов на расширение своего ассортимента. Часть денег была взята под 7%, часть — под 8%, часть — под 10%. Сумма займа под 10% в четыре раза превышала сумму займа под 7%, а годовая процентная ставка по всем трем займам составляла 130 500 долларов. Используйте матрицы, чтобы найти сумму, заимствованную по каждой ставке.

    Показать решение

    150 000 долларов США под 7%, 750 000 долларов США под 8%, 600 000 долларов США под 10%

    Внесите свой вклад!

    У вас была идея улучшить этот контент? Нам очень понравится ваш вклад.

    Улучшить эту страницуПодробнее

    систем линейных уравнений: исключение Гаусса

    систем линейных уравнений: исключение Гаусса

    Решать нелинейные системы уравнений довольно сложно, а линейные системы довольно легко изучать. Существуют численные методы, которые помогают аппроксимировать нелинейные системы линейными в надежде, что решения линейных систем достаточно близки к решениям нелинейных систем. Мы не будем здесь обсуждать это.Вместо этого мы сосредоточим наше внимание на линейных системах.

    Для простоты мы ограничимся тремя, максимум четырьмя неизвестными. Читатель, интересующийся случаем большего количества неизвестных, может легко развить следующие идеи.

    Определение. Уравнение

    a x + b y + c z + d w = h


    где a , b , c , d и h — известные числа, а x , y , z и w — неизвестные числа. называется линейным уравнением .Если h = 0, линейное уравнение называется однородным . Линейная система представляет собой набор линейных уравнений, а однородная линейная система представляет собой набор однородных линейных уравнений.

    Например,


    а также

    линейные системы, а

    является нелинейной системой (из-за y 2 ). Система

    является однородной линейной системой.

    Матричное представление линейной системы

    Матрицы помогают переписать линейную систему в очень простой форме.Затем для решения систем можно использовать алгебраические свойства матриц. Сначала рассмотрим линейную систему


    Установите матрицы

    Используя матричное умножение, мы можем переписать линейную систему выше как матричное уравнение

    Как видите, это намного лучше, чем уравнения. Но иногда стоит решить систему напрямую, минуя матричную форму. Матрица A называется матричным коэффициентом линейной системы.Матрица C называется неоднородным членом . Когда , линейная система однородна. Матрица X — это неизвестная матрица. Его записи являются неизвестными линейной системы. Расширенная матрица , связанная с системой, является матрицей [ A | C ], где

    В общем, если линейная система имеет n уравнений с m неизвестными, то матричный коэффициент будет матрицей nxm, а расширенная матрица — матрицей nx (m + 1).Теперь обратим внимание на решения системы.

    Определение. Две линейные системы с n неизвестными считаются эквивалентными тогда и только тогда, когда они имеют одинаковый набор решений.

    Это определение важно, поскольку идея решения системы состоит в том, чтобы найти эквивалентную систему, которую легко решить. Вы можете спросить, как мы сможем создать такую ​​систему? Легко, мы делаем это с помощью элементарных операций . Действительно, ясно, что если мы поменяем местами два уравнения, новая система все равно будет эквивалентна старой.Если мы умножим уравнение на ненулевое число, мы получим новую систему, по-прежнему эквивалентную старой. И, наконец, заменив одно уравнение суммой двух уравнений, мы снова получим эквивалентную систему. Эти операции называются элементарными операциями в системах. Посмотрим, как это работает в конкретном случае.

    Пример. Рассмотрим линейную систему

    Идея состоит в том, чтобы сохранить первое уравнение и поработать над двумя последними. При этом мы попытаемся убить одного из неизвестных и решить два других.Например, если мы сохраним первое и второе уравнение и вычтем первое из последнего, мы получим эквивалентную систему


    Затем мы сохраняем первое и последнее уравнение и вычитаем первое из второго. Получаем эквивалентную систему

    Теперь мы сосредоточимся на втором и третьем уравнениях. Повторяем ту же процедуру. Попробуйте убить одного из двух неизвестных ( y или z ). Действительно, мы сохраняем первое и второе уравнение и добавляем второе к третьему, умножив его на 3.Мы получили

    Это, очевидно, означает z = -2. Из второго уравнения мы получаем y = -2, и, наконец, из первого уравнения мы получаем x = 4. Следовательно, линейная система имеет одно решение.

    Переход от последнего уравнения к первому при решении для неизвестных называется обратным решением .

    Имейте в виду, что линейные системы, для которых матричный коэффициент является верхнетреугольным, легко решить. Это особенно верно, если матрица имеет эшелонированную форму.Таким образом, фокус состоит в том, чтобы выполнить элементарные операции по преобразованию исходной линейной системы в другую, для которой матрица коэффициентов имеет эшелонированную форму.
    Используя наши знания о матрицах, можем ли мы в любом случае переписать то, что мы сделали выше, в матричной форме, которая упростит нашу нотацию (или представление)? Действительно, рассмотрим расширенную матрицу


    Выполним над этой матрицей несколько элементарных операций со строками. Действительно, если мы сохраним первую и вторую строки и вычтем первую из последней, мы получим

    Затем мы сохраняем первую и последнюю строки и вычитаем первую из второй.Мы получили

    Затем мы сохраняем первую и вторую строки и добавляем вторую к третьей, умножив ее на 3, чтобы получить

    Это треугольная матрица, не имеющая эшелонированной формы. Линейная система, для которой эта матрица является расширенной, есть

    Как видите, мы получили ту же систему, что и раньше. Фактически мы следовали тем же элементарным операциям, что и выше. На каждом этапе новая матрица была в точности расширенной матрицей, связанной с новой системой.Это показывает, что вместо того, чтобы писать системы снова и снова, легко поиграться с элементарными операциями со строками, и как только мы получим треугольную матрицу, напишем связанную линейную систему, а затем решим ее. Это известно как исключения по Гауссу . Подведем итоги процедуры:

    Исключение Гаусса. Рассмотрим линейную систему.

    1.
    Построить расширенную матрицу для системы;
    2.
    Используйте элементарные операции со строками, чтобы преобразовать расширенную матрицу в треугольную;
    3.
    Запишите новую линейную систему, для которой треугольная матрица является связанной с ней расширенной матрицей;
    4.
    Решите новую систему. Вам может потребоваться присвоить некоторые параметрические значения некоторым неизвестным, а затем применить метод обратной подстановки для решения новой системы.

    Пример. Решите следующую систему методом исключения Гаусса


    Расширенная матрица

    Мы используем элементарные операции со строками, чтобы преобразовать эту матрицу в треугольную.Мы сохраняем первую строку и используем ее для получения всех нулей в любом месте первого столбца. У нас есть

    Далее мы сохраняем первую и вторую строки и стараемся, чтобы во втором столбце были нули. Мы получили

    Далее сохраняем первые три ряда. Добавляем последний к третьему, чтобы получить

    Это треугольная матрица. Связанная с ним система

    Очевидно, что v = 1. Установите z = s и w = t , тогда мы имеем

    Из первого уравнения следует Используя алгебраические манипуляции, получаем
    x = — — с т .
    Собрав все вместе, у нас есть

    Пример. Используйте метод исключения Гаусса для решения линейной системы


    Соответствующая расширенная матрица

    Сохраняем первую строку и вычитаем первую строку, умноженную на 2, из второй строки. Мы получили

    Это треугольная матрица. Связанная система

    Ясно, что второе уравнение означает, что эта система не имеет решения. Следовательно, эта линейная система не имеет решения.

    Определение. Линейная система называется непоследовательной или переопределенной , если у нее нет решения. Другими словами, набор решений пуст. В противном случае линейная система называется согласованной .

    Следуя приведенному выше примеру, мы видим, что если мы выполним элементарные операции со строками над расширенной матрицей системы и получим матрицу с одной из строк, равной , где , тогда система несовместима.

    [Назад] [Следующий] [Геометрия] [Алгебра] [Тригонометрия] [Исчисление] [Дифференциальные уравнения] [Матричная алгебра]

    С.O.S MATH: Домашняя страница

    Вам нужна дополнительная помощь? Пожалуйста, разместите свой вопрос на нашем S.O.S. Математика CyberBoard.

    Автор : М.А.Хамси

    Авторские права 1999-2021 MathMedics, LLC. Все права защищены.
    Свяжитесь с нами
    Math Medics, LLC. — П.О. Box 12395 — El Paso TX 79913 — США
    пользователей онлайн за последний час

    Исключение Гаусса — обзор

    1.3.7 Исключение Гаусса или Гаусса

    Исключение Гаусса (также известное как Исключение Гаусса ) — широко используемый метод для решения систем линейных уравнений в форме [ K ] { u } = { F }.В операциях с матрицами существует три распространенных типа манипуляций, которые служат для создания новой матрицы, обладающей теми же характеристиками, что и исходная:

    1.

    Поменять местами любые две строки.

    2.

    Умножьте каждую запись в любой строке на ненулевое постоянное значение.

    3.

    Добавьте значения из каждой записи одной строки к каждой записи другой строки.

    Цель использования исключения Гаусса — создать новую матрицу с теми же свойствами, что и исходная [ K ], но в формате, в котором только верхний треугольник имеет ненулевые элементы.Используя предыдущую матрицу 5 × 5 в качестве примера, верхний треугольник состоит из элементов в правом верхнем треугольнике матрицы и включает элементы в правой диагональной строке в виде

    [m11m12m13m14m150m22m23m24m2500m33m34m35000m44m450000m55].

    Мы достигаем цели исключения Гаусса, правильно применяя одну из трех вышеупомянутых операций за раз. После того, как верхняя треугольная матрица сформирована, мы используем метод обратной подстановки , чтобы сначала найти последнюю переменную.Причина, по которой этот метод называется «обратной заменой», заключается в том, что последняя строка верхней треугольной матрицы должна быть решена первой. Поскольку в последней строке верхней треугольной матрицы есть только одна ненулевая запись, мы можем найти неизвестную переменную простым арифметическим делением, то есть из

    [K] {u2v2u3u4v4} = [m11m12m13m14m150m22m23m24m2500m33m34m35000m44m450000m55] {u4 } → v4 = F4Vm55.

    Имея значение v 4 , мы решаем от второй до последней переменной.Поскольку m 44 u 4 + m 45 v 4 = F 4 H , мы можем решить для u 4 F4H − m45v4m44. Мы многократно применяем один и тот же набор процедур, пока не будут найдены значения всех переменных.

    Мы будем использовать типичный 64-битный компьютер, чтобы проиллюстрировать критическую проблему при использовании исключения Гаусса. Хорошо известно, что такой компьютер хранит действительное (десятичное) число в формате с плавающей запятой, используя 64 бита: 1 бит для представления знака (плюс или минус), 52 бита для представления числа точных цифр (мантисса), и 11 бит для представления экспоненты.При делении числа на другое очень маленькое число имеющихся цифр в мантиссе может быть недостаточно для поддержания необходимой точности, то есть может возникнуть ошибка округления. В исключении Гаусса точка поворота или позиция поворота — это позиция в строке, которая совпадает с правой диагональной линией. Значения в точках поворота используются в качестве знаменателя при формировании верхней треугольной матрицы. Чтобы исключить ошибки округления, возникающие при делении на очень маленькое число, используется первый тип манипуляции для перемещения строки с очень маленьким числом в точке поворота в другую строку.Это достигается простым перестановкой рядов так, чтобы большие числа располагались в точках поворота. Вторую и третью операции мы используем для получения нулей в левой нижней части матрицы, что необходимо для получения верхней треугольной матрицы.

    Модифицированной версией метода исключения Гаусса является метод исключения Гаусса – Жордана. Цель исключения Гаусса – Жордана — получить матрицу, которая имеет правую диагональную линию всех единиц (единиц), а все остальные позиции матрицы содержат нули.Это достигается с помощью тех же трех типов манипуляций с матрицами, которые используются в методе исключения Гаусса. Поскольку квадратная матрица состоит только из единичных значений в диагональных элементах, решения для всех неизвестных становятся легко доступными. Один из недостатков метода Гаусса – Жордана заключается в том, что он более затратный в вычислительном отношении, чем метод исключения Гаусса. Таким образом, он полезен только для решения проблем путем ручного расчета, когда существует небольшое количество одновременных уравнений.Используя метод исключения Гаусса, а не метод Гаусса – Жордана, мы избегаем многих дополнительных шагов. Поскольку метод FE обычно включает большую систему, чаще используется метод исключения Гаусса.

    В следующем разделе мы шаг за шагом продемонстрируем процессы в методе исключения Гаусса. Конечно, вместо ручных вычислений следует написать и использовать компьютерную программу. Используя предыдущий пример в качестве отправной точки, уравнение. (1.68) повторяется ниже.

    [K] {u2v2u3u4v4} = 108 [100−50006.6700−6.67−506.44−1.441.9200−1.44400−6.671.

    .67] {u2v2u3u4v4} = {F2HF2VF3HF4HF4−5002} = {00020000 =

    002 все, кроме первой записи в первом столбце, равны 0. Мы заметили, что третья строка в этом столбце содержит единственное ненулевое значение. Чтобы манипулировать третьей строкой, чтобы сделать ведущее число 0, мы должны умножить существующее число (-5) на такое значение, чтобы добавление результата к первой записи в строке один (10) давало 0. Используя правило два, мы умножаем каждая запись в третьей строке по 2:

    108 [100-50006.6700−6.67−10012.88−2.883.8400−1.44400−6.671.

    .67] {u2v2u3u4v4} = {00020000−50000}.

    Затем мы добавляем строку 1 к строке 3, но мы не затрагиваем строку 1:

    108 [100-50006.6700-6.67007.88-2.883.8400-1.44400-6.671.

    .67] {u2v2u3u4v4} = {00020000-50000 }.

    Теперь, когда все значения в первом столбце, кроме первого, равны 0, мы применяем аналогичный процесс ко второму столбцу. Мы хотим, чтобы все значения во втором столбце, кроме второго, равнялись 0, а это значит, что мы должны адресовать −6,67 в последней строке. Его можно изменить на 0, просто добавив значения из строки 2.

    108 [100−50006.6700−6.67007.88−2.883.8400−1.4440001.9204] {u2v2u3u4v4} = {00020000−50000}

    Две операции, правила два и три, необходимы для преобразования записи в четвертой строке столбца три на 0. Сначала умножаем четвертую строку на 7,881,44 (обратите внимание, что эта операция также применима к вектору силы):

    108 [100-50006,6700-6,67007,88-2,883,8400-7,8821,8,9204] {u2v2u3u4v4} = { 0001.094 × 105−50000},

    , а затем добавьте значения из третьей строки к этим результатам, чтобы сформировать новую четвертую строку:

    108 [100−50006.6700−6.67007.88−2.883.8400019.013.84001.9204] {u2v2u3u4v4} = {0001.094 × 105−50000}.

    Аналогичным образом мы умножаем пятую строку на -7,881,92, а затем складываем значения из третьей строки, чтобы сформировать новую пятую строку:

    108 [100-50006,6700-6,67007,88-2,883,8400019,013,84000-2,88-12,58 ] {u2v2u3u4v4} = {0001,094 × 1052,052 × 106}.

    К этому моменту должно быть очевидно, что умножение на значение в другой строке и последующее деление на значение в текущей строке дает результат, который можно вычесть из этой другой строки и получить 0.Чтобы еще раз увидеть этот процесс, мы умножаем пятую строку на 19.012.88, затем складываем значения из четвертой строки, чтобы получить новую пятую строку:

    108 [100−50006.6700−6.67007.88−2.883.8400019.013.840000−79.20] {u2v2u3u4v4} = {0001,094 × 1051,464 × 106}.

    Теперь матрица имеет форму верхней треугольной матрицы, что означает, что все значения ниже и слева от правой диагональной линии являются нулями. На этом этапе мы применяем метод обратной замены для определения узловых смещений.

    Начнем с последней строки, которая содержит 108 [0000−79.20], и мы умножаем последовательные значения в этой строке на последовательные значения в векторе узлового смещения:

    108 ((0) (u2) + (0) (v2) + (0) (u3) + (0) ( u4) + (- 79.20) (v4)) = 1.464 × 106

    Мы можем сделать это проще, признав, что только последнее значение в строке ненулевое, и поэтому v 4 — это просто последнее значение в вектор силы, деленный на последний элемент в верхней треугольной матрице [ K ]:

    v4 = (1,464 × 106) (- 79,2 × 108) = — 1,849 × 10−4

    Мы можем использовать v 4 , чтобы найти u 4 и т. Д.Ниже приведены расчеты значений узловых смещений в м :

    u4 = 1,094 × 105−108 × 3,84 × v419,01 × 108 = 1,804 × 10519,01 × 108 = 0,949 × 10−4,

    u3 = 108 × (2,88) × u4−108 × 3,84 × v4108 × 7,88 = 9,837 × 10−47,88 = 1,248 × 10−4,

    v2 = 108 × 6,67 × v4108 × 6,67 = −1,849 × 10−4 и

    u2 = 108 × 5 × u3108 × 10 = 0,624 × 10−4.

    Узловые смещения, рассчитанные с использованием метода MSA или прямого метода жесткости, в точности совпадают с точными решениями проблем, связанных с фермами.Для типов элементов, отличных от фермы или пружины, узловые решения вряд ли будут иметь те же значения, что и точные решения. Простое практическое правило состоит в том, что чем больше элементов используется для представления интересующей структуры, тем точнее результаты будут приближаться к точным решениям. Дополнительные описания других типов элементов приведены в главе 2.

    Исключение Гаусса Джордана — объяснение и примеры

    Метод методом исключения Гаусса-Жордана — это алгоритм для решения линейной системы уравнений.Мы также можем использовать его, чтобы найти обратную матрицу. Давайте сначала посмотрим на определение:

    Исключение Гаусса Джордана или Гаусса исключение — это алгоритм для решения системы линейных уравнений, представляющий ее в виде расширенной матрицы, сокращая ее с помощью операций со строками и выражая систему в сокращенной строке. -эшелонированная форма для нахождения значений переменных.

    В этом уроке мы увидим детали метода исключения Гаусса и того, как решить систему линейных уравнений с помощью метода исключения Гаусса-Жордана.Примеры и практические вопросы будут приведены ниже.

    Что такое метод исключения Гаусса?

    Метод исключения Гаусса — это структурированный метод решения системы линейных уравнений. Таким образом, это алгоритм, и его можно легко запрограммировать для решения системы линейных уравнений. Основная цель исключения Гаусса-Джордана:

    • представить систему линейных уравнений в форме расширенной матрицы
    • затем выполнить операции строки $ 3 $ до тех пор, пока не будет получена сокращенная форма эшелона строк (RREF) достигнуто
    • Наконец, мы можем легко распознать решения из RREF

    . Давайте посмотрим, что такое расширенная матричная форма, операции со строками стоимостью 3 доллара, которые мы можем выполнять с матрицей, и упрощенная форма эшелона строк матрицы.

    Расширенная матрица

    Система линейных уравнений показана ниже:

    $ \ begin {align *} 2x + 3y & = \, 7 \\ x — y & = 4 \ end {align *} $

    Мы запишет расширенную матрицу этой системы, используя коэффициенты уравнений и записав ее в стиле , показанном ниже:

    $ \ left [\ begin {array} {rr | r} 2 & 3 & 7 \\ 1 & -1 & 4 \ end {array} \ right] $

    Пример использования одновременных уравнений $ 3 $ показан ниже:

    $ \ begin {align *} 2x + y + z & = \, 10 \\ x + 2y + 3z & = 1 \\ — x — y — z & = 2 \ end {align *} $

    Представление этой системы в виде расширенной матрицы:

    $ \ left [\ begin {array} {rrr | r} 2 & 1 & 1 & 10 \\ 1 & 2 & 3 & 1 \\ — 1 & — 1 & — 1 & 2 \ end {array} \ right] $

    Операции со строками в матрице

    Есть $ 3 $ элементарных операций со строками , которые мы можем выполнять с матрицами.Это не изменит решения системы. Это:

    1. Обмен $ 2 $ строк
    2. Умножить строку на ненулевой ($ \ neq 0 $) скаляр
    3. Добавить или вычесть скалярное кратное одной строки из другой строки.

    Форма сокращенного эшелона строк

    Основная цель исключения Гаусса Джордана — использовать операции элементарной строки стоимостью 3 доллара в расширенной матрице, чтобы привести ее к форме сокращенного эшелона строк (RREF). Считается, что матрица находится в сокращенной форме эшелона строк , также известной как каноническая форма строки , если выполняются следующие условия $ 4 $:

    1. Строки с нулевыми записями (все элементы этой строки равны $ 0 $ s) находятся внизу матрицы.
    2. ведущая запись (первая ненулевая запись в строке) каждой ненулевой строки соответствует правой ведущей записи строки непосредственно над ней.
    3. Начальная запись в любой ненулевой строке — 1 доллар.
    4. Все записи в столбце, содержащем начальную запись ($ 1 $), нулевые.

    Как выполнить исключение Гаусса-Джордана

    В методе исключения Гаусса-Джордана нет каких-либо определенных шагов, но алгоритм ниже описывает шаги, которые мы выполняем, чтобы прийти к сокращенной форме эшелона строк расширенной матрицы.

    1. Поменяйте местами строки так, чтобы все строки с нулевыми записями находились внизу матрицы.
    2. Поменяйте местами строки так, чтобы строка с самой большой левой цифрой находилась наверху матрицы.
    3. Умножьте верхнюю строку на скаляр, который преобразует ведущую запись верхней строки в $ 1 $ (если ведущей записью верхней строки является $ a $, умножьте ее на $ \ frac {1} {a} $, чтобы получить $ 1 $).
    4. Сложите или вычтите значения, кратные верхней строке, из других строк, чтобы все записи в столбце ведущей записи верхней строки были нулями.
    5. Выполните шаги $ 2 — 4 $ для следующей крайней левой ненулевой записи , пока все ведущие записи каждой строки не будут равны 1 $.
    6. Поменяйте местами строки так, чтобы первая запись каждой ненулевой строки находилась справа от ведущей записи строки непосредственно над ней

    На первый взгляд, запомнить / запомнить шаги не так просто. Это вопрос решения нескольких проблем, пока вы не освоитесь с процессом. Существует также фактор интуиции , который играет B-I-G роль в выполнении исключения Гаусса Джордана.

    Давайте рассмотрим несколько примеров, чтобы пояснить процесс решения системы линейных уравнений с помощью метода исключения Гаусса-Джордана .

    Пример 1

    Решите систему, показанную ниже, используя метод исключения Гаусса Джордана:

    $ \ begin {align *} {- x} + 2y & = \, {- 6} \\ { 3x} — 4y & = {14} \ end {align *} $

    Решение

    Первый шаг — написать расширенную матрицу системы.Мы показываем это ниже:

    $ \ left [\ begin {array} {r r | r} — 1 & 2 & — 6 \\ 3 & -4 & 14 \ end {array} \ right] $

    Теперь наша задача — преобразовать матрицу в сокращенную форму эшелона строк (RREF), выполнив команду $ 3 $ элементарные операции со строками.

    У нас есть расширенная матрица:

    $ \ left [\ begin {array} {r r | r} — 1 & 2 & — 6 \\ 3 & — 4 & 14 \ end {array} \ right] $

    Шаг 1:

    Мы можем умножить первую строку на $ — 1 $, чтобы получить ведущий вход $ 1 $.Показано ниже:

    $ \ left [\ begin {array} {r r | r} 1 & — 2 & 6 \\ 3 & — 4 & 14 \ end {array} \ right] $

    Шаг 2:

    Теперь мы можем умножить первую строку на $ 3 $ и вычесть ее из второй ряд. Показано ниже:

    $ \ left [\ begin {array} {r r | r} 1 & -2 & 6 \\ {3 — (1 \ times 3)} & {-4 — (-2 \ times 3)} & {14 — (6 \ times 3)} \ end {array} \ справа] $

    $ = \ left [\ begin {array} {rr | r} 1 & — 2 & 6 \\ 0 & 2 & — 4 \ end {array} \ right] $

    У нас есть $ 0 $ как первая запись второй строки.

    Шаг 3:

    Чтобы сделать вторую запись второй строки $ 1 $, мы можем умножить вторую строку на $ \ frac {1} {2} $. Показано ниже:

    $ \ left [\ begin {array} {r r | r} 1 & — 2 & 6 \\ {\ frac {1} {2} \ times 0} & {\ frac {1} {2} \ times 2} & {\ frac {1} {2} \ times — 4} \ end {array} \ right] $

    $ = \ left [\ begin {array} {rr | r} 1 & — 2 & 6 \\ 0 & 1 & — 2 \ end {array} \ right] $

    Шаг 4:

    Мы почти у цели!

    Вторая запись первой строки должна быть $ 0 $.Для этого мы умножаем вторую строку на $ 2 $ и добавляем ее к первой строке. Показано ниже:

    $ \ left [\ begin {array} {r r | r} {1 + (0 \ times 2)} & {- 2 + (1 \ times 2)} & {6 + (- 2 \ times 2)} \\ 0 & 1 & — 2 \ end {array} \ справа] $

    $ = \ left [\ begin {array} {rr | r} 1 & 0 & 2 \\ 0 & 1 & — 2 \ end {array} \ right] $

    Это сокращенный эшелон строк , форма . Из расширенной матрицы мы можем написать два уравнения (решения):

    $ \ begin {align *} x + 0y & = \, 2 \\ 0x + y & = -2 \ end {align *} $

    $ \ begin {align *} x & = \, 2 \\ y & = — 2 \ end {align *} $

    Таким образом, решение системы уравнений: $ x = 2 $ и $ y = — 2 $.

    Пример 2

    Решите систему, показанную ниже, используя метод исключения Гаусса Джордана:

    $ \ begin {align *} x + 2y & = \, 4 \\ x — 2y & = 6 \ end { align *} $


    Решение

    Запишем расширенную матрицу системы уравнений:

    $ \ left [\ begin {array} {rr | r} 1 & 2 & 4 \\ 1 & — 2 & 6 \ end {array} \ right] $

    Теперь мы выполняем элементарные операции со строками над этой матрицей, пока не получим сокращенную форму эшелона строк.

    Шаг 1:

    Умножаем первую строку на $ 1 $, а затем вычитаем ее из второй строки. По сути это вычитание первой строки из второй:

    $ \ left [\ begin {array} {r r | r} 1 и 2 и 4 \\ 1 — 1 и — 2 — 2 и 6 — 4 \ end {array} \ right] $

    $ = \ left [\ begin {array} {r r | r} 1 & 2 & 4 \\ 0 & — 4 & 2 \ end {array} \ right] $

    Шаг 2:

    Мы умножаем вторую строку на $ — \ frac {1} {4} $, чтобы получить вторая запись строки, $ 1 $:

    $ \ left [\ begin {array} {rr | r} 1 и 2 и 4 \\ 0 \ times — \ frac {1} {4} & — 4 \ times — \ frac {1} {4} и 2 \ times — \ frac {1} {4} \ end {массив} \ right] $

    $ = \ left [\ begin {array} {rr | r} 1 & 2 & 4 \\ 0 & 1 & — \ frac {1} {2} \ end {array} \ right] $

    Шаг 3:

    Наконец, мы умножаем вторую строку на $ — 2 $ и добавьте его в первую строку, чтобы получить уменьшенную форму эшелона строк этой матрицы:

    $ \ left [\ begin {array} {rr | r} 1 + (- 2 \ times 0) & 2+ (- 2 \ times 1) & 4 + (- 2 \ times — \ frac {1} {2}) \\ 0 & 1 & — \ frac {1 } {2} \ end {array} \ right] $

    $ = \ left [\ begin {array} {rr | r} 1 & 0 & 5 \\ 0 & 1 & — \ frac {1} {2} \ end {array} \ right] $

    Это сокращенный эшелон строки , форма .Из расширенной матрицы мы можем написать два уравнения (решения):

    $ \ begin {align *} x + 0y & = \, 5 \\ 0x + y & = — \ frac {1} {2} \ end {align *} $

    $ \ begin {align *} x & = \, 5 \\ y & = — \ frac {1} {2} \ end {align *} $

    Таким образом, решение системы уравнений составляет $ x = 5 $ и $ y = — \ frac {1} {2} $.

    Практические вопросы
    1. Решите систему, показанную ниже, используя метод исключения Гаусса Джордана:

      $ \ begin {align *} 2x + y & = \, — 3 \\ — x — y & = 2 \ end {align *} $

    2. Решите систему, показанную ниже, используя метод исключения Гаусса Джордана:

      $ \ begin {align *} x + 5y & = \, 15 \\ — x + 5y & = 25 \ end {align *} $

    Ответы

    1. Начнем с написания расширенной матрицы системы уравнений:

      $ \ left [\ begin {array} {rr | r} 2 & 1 & — 3 \\ — 1 & — 1 & 2 \ end {array} \ right] $

      Теперь мы выполняем элементарные операции со строками, чтобы прийти к нашему решению.

      Первый,
      Инвертируем знаки второй строки и меняем строки местами. Итак, имеем:
      $ \ left [\ begin {array} {r r | r} 1 & 1 & — 2 \\ 2 & 1 & — 3 \ end {array} \ right] $
      Во-вторых,
      Мы дважды вычитаем первую строку из второй строки:
      $ \ left [\ begin {array} { rr | r} 1 & 1 & — 2 \\ 2 — (2 \ times 1) & 1 — (2 \ times 1) & — 3 — (2 \ times — 2) \ end {array} \ right] $
      $ = \ left [\ begin {array} {rr | r} 1 & 1 & — 2 \\ 0 & — 1 & 1 \ end {array} \ right] $
      В-третьих,
      Мы инвертируем вторую строку, чтобы получить:
      $ = \ left [\ begin {array} {rr | r} 1 & 1 & — 2 \\ 0 & 1 & — 1 \ end {array} \ right] $
      Наконец,
      Мы вычитаем вторую строку из первой и получаем:
      $ = \ left [\ begin { массив} {rr | r} 1 & 0 & — 1 \\ 0 & 1 & — 1 \ end {array} \ right] $

      Из этой расширенной матрицы мы можем написать два уравнения (решения):

      $ \ begin {align *} x + 0y & = \, — 1 \\ 0x + y & = — 1 \ end {align *} $

      $ \ begin {align *} x & = \, — 1 \\ y & = — 1 \ end {align *} $

      Таким образом, решение системы уравнений: $ x = — 1 $ и $ y = — 1 $.

    2. Расширенная матрица системы:
      $ \ left [\ begin {array} {rr | r} 1 & 5 & 15 \\ — 1 & 5 & 25 \ end {array} \ right] $
      Давайте приведите эту матрицу к приведенной форме эшелона строк и найдите решение системы.

      Во-первых,
      Отмените первую строку, затем вычтите ее из второй строки, чтобы получить:
      $ \ left [\ begin {array} {rr | r} 1 & 5 & 15 \\ — 1 — (- 1) & 5 — (- 5) & 25 — (- 15) \ end {array} \ right] $
      $ = \ left [\ begin {array} {rr | r} 1 & 5 & 15 \\ 0 & 10 & 40 \ end {array} \ right] $
      Second,
      Разделите вторую строку на $ 10 $, чтобы получить:
      $ \ left [\ begin {array} {rr | r} 1 & 5 & 15 \\ 0 & 1 & 4 \ end {array} \ right] $
      Затем
      Умножьте вторую строку на $ 5 $ и вычтите ее из первой строки, чтобы получить окончательное решение:
      $ \ left [\ begin {array} {rr | r} 1 — (5 \ times 0) & 5 — (5 \ times 1) & 15 — (5 \ times 4) \\ 0 & 1 & 4 \ end {array} \ right] $
      $ = \ left [ \ begin {array} {rr | r} 1 & 0 & — 5 \\ 0 & 1 & 4 \ end {array} \ right] $
      Это сокращенная форма эшелона строк (RREF).Из этой расширенной матрицы мы можем написать два уравнения (решения):

      $ \ begin {align *} x & = \, — 5 \\ y & = 4 \ end {align *} $

      Таким образом, решение системы уравнений $ x = — 5 $ и $ y = 4 $.

    Предыдущий урок | Главная страница | Следующий урок

    Исключение Гаусса (Введение в линейные системы)

    Метод исключения Гаусса — это процедура решения систем линейных уравнений. Его можно описать как последовательность операций, выполняемых над соответствующей матрицей коэффициентов.Мы мотивируем исключение Гаусса и исключение Гаусса – Жордана несколькими примерами с упором на понимание операций со строками.

    Введение в системы линейных уравнений

    Система линейных уравнений с двумя переменными $ x $ и $ y $ имеет вид $$ \ begin {cases} ax + by = c \\ d x + ey = f \ end {ases} $$ где $ a, b , c, d, e, f $ заданы числа, такие как действительные или комплексные числа. Иногда система линейных уравнений также записывается с использованием индексов \ begin {equal} \ label {2by2sys} \ begin {cases} a_ {11} x + a_ {12} y = b_1 \\ a_ {21} x + a_ {22} y = b_2 \ end {case} \ end {формула}, чтобы уменьшить количество используемых букв.Для простоты давайте временно предположим, что коэффициенты являются ненулевыми действительными числами, и зададимся вопросом: сколько решений может быть у системы $ 2 \ times 2 $? Ключевая идея состоит в том, чтобы понять, что каждое линейное уравнение представляет собой линию в декартовой плоскости. Если мы рассмотрим возможные способы пересечения прямых на плоскости, мы приходим к выводу, что не должно быть решений, одно единственное решение или бесконечно много точек $ (x, y) $, которые решают систему.

    Пример .Определите, соответствует ли система линейных уравнений \ begin {уравнение} \ label {consys} \ begin {cases} 2x + 3y = 0 \\ 4x + 5y = 0. \ end {ases} \ end {Equation} не имеет решений, ровно одно решение или бесконечно много решений. Если мы умножим первое уравнение, а именно $ 2x + 3y = 0 $, на 2 и вычтем из второго уравнения $ 4x + 5y = 0 $, мы получим $ y = 0. $ Следовательно, решение является единственным и имеет вид $ (x , y) = (0,0). $

    Система линейных уравнений $ 3 \ times 3 $ имеет вид \ begin {уравнение} \ label {3by3sys} \ begin {cases} a_ {11} x + a_ {12} y + a_ {13} z = b_1 \\ a_ {21} x + a_ {22} y + a_ {23} z = b_2 \\ a_ {31} x + a_ {32} y + a_ {33} z = b_3 \\ \ end {case} \ end { уравнение}, где $ a_ {ij} $ и $ b_1, b_2, b_3 $ — числа, а $ x, y, z $ — переменные.Геометрически линейные уравнения с тремя переменными — это просто плоскости в трех измерениях. Итак, каковы различные типы наборов решений для системы? Рассматривая возможные способы пересечения трех плоскостей в трех измерениях, мы приходим к выводу, что не должно быть решений, одно единственное решение или бесконечно много точек $ (x, y, z) $, которые решают систему.

    Пример . Определите, соответствует ли система линейных уравнений \ begin {уравнение} \ label {linesysex1} \ begin {cases} x + 2y + 3z = 0 \\ 4x + 5y + 6z = 3 \\ 7x + 8y + 9z = 0 \ end { case} \ end {equal} не имеет решений, ровно одно решение или бесконечно много решений.Умножьте первое уравнение на $ -2 $ и добавьте ко второму уравнению, получив уравнение $ 2x + y = 3. $ Удалив $ z $ из первого и третьего уравнений, мы получим $ 4x + 2y = 0 $, умножив первое уравнение. на $ -3 $ и добавив к третьему уравнению. $ X $ и $ y $, удовлетворяющие системе, также должны удовлетворять системе \ begin {уравнение} \ begin {cases} 2x + y = 3 \\ 4x + 2y = 0. \ end {ases} \ end {Equation} Умножение первого уравнения на $ 2 $ дает $ 4x + 2y = 6. $ Обратите внимание, что нет $ x $ и $ y $, которые удовлетворяют как $ 4x + 2y = 6 $, так и $ 4x + 2у = 0.$ Таким образом, у системы нет решений; поэтому исходная система также не имеет решений.

    Пример . Для чисел $ a, b $ и $ c $ покажите, что система линейных уравнений $$ \ begin {cases} x + 2y + 3z = a \\ x + 3y + 8z = b \\ x + 2y + 2z = c \ end {case} $$ либо не имеет решений, либо ровно одно решение, либо бесконечно много решений. Мы выбираем сначала удалить $ x $ и получаем систему $$ \ begin {cases} -y-5z = a-b \\ z = a-c. \ end {ases} $$ Затем мы исключаем $ z $, получая $ y = -6a + b + 5c.$ Следовательно, единственное решение — $$ (x, y, z) = (10a-2b-7c, -6a + b + 5c, a-c). $

    Давайте рассмотрим набор линейных уравнений, который включает $ n $ неизвестных величин, представленных $ x_1, x_2, \ ldots, x_n. $ Пусть $ a_ {ij} $ представляет число, которое является коэффициентом $ x_j $ в $ i $ -ое уравнение. Пусть даны числа $ b_1, b_2, \ ldots, b_m $. Система линейных уравнений уравнений \ begin {equal} \ label {syseq} \ begin {cases} a_ {11} x_1 + a_ {12} x_2 + \ cdots + a_ {1n} x_n = b_1 \\ a_ {21} x_1 + a_ {22} x_2 + \ cdots + a_ {2n} x_n = b_2 \\ \ hfill \ vdots \ hfill \\ a_ {m1} x_1 + a_ {m2} x_2 + \ cdots + a_ {mn} x_n = b_m \\ \ end {cases} \ end {Equation} называется системой одновременных линейных алгебраических уравнений .

    Решение этой системы — это упорядоченный набор из $ n $ чисел, который удовлетворяет каждому из операторов $ m $ в системе. Система линейных уравнений без решения называется несовместимой , а система по крайней мере с одним решением называется согласованной . Массив $$ \ left [\ begin {array} {l | l} \ begin {matrix} a_ {11} & a_ {12} & \ cdots & a_ {1n} \\ a_ {21} & a_ {22} & \ cdots & a_ {2n} \\ & & \ vdots \\ a_ {m1} & a_ {m2} & \ cdots & a_ {mn} \\ \ end {matrix} & \ begin {matrix} b_1 \\ b_2 \\ \ vdots \\ b_m \ end {matrix} \ end {array} \ right] $$ называется расширенной матрицей , соответствующей системе линейных уравнений.

    Например, система линейных уравнений выше оказалась непротиворечивой, а другая система линейных уравнений выше оказалась непоследовательной. Дополненные матрицы для этих систем следующие. $$ \ left [\ begin {array} {l | l} \ begin {matrix} 2 & 3 \\ 4 & 5 \ end {matrix} & \ begin {matrix} 0 \\ 0 \ end {matrix} \ end {массив} \ right] \ qquad \ text {and} \ qquad \ left [\ begin {array} {l | l} \ begin {matrix} 1 & 2 & 3 \\ 4 & 5 & 6 \\ 7 & 8 & 9 \ end {matrix} & \ begin {matrix} 0 \\ 3 \\ 0 \ end {matrix} \ end {array} \ right] $$

    Пример .Найдите все решения следующей системы линейных уравнений. \ begin {Equation} \ label {example: 2by3system} \ begin {cases} -150 x + 500y = z \\ 50x + 100y + z = 200 \ end {cases} \ end {equal} Данная система эквивалентна $ $ \ begin {case} -150 x + 500y-z = 0 \\ 50x + 100y + z = 200. \ end {ases} $$ Сложение этих уравнений дает $ -100x + 600y = 200. $ Поскольку у нас есть одно уравнение с двумя переменными, одна из переменных свободна. Мы решили позволить $ y $ быть свободным. Пусть $ y = t $ для произвольного числа $ t. $ Тогда, решая для $ x $, мы получаем $ -100x = 200y-600y $, или, что то же самое, $ x = -2 + 6t.$ Подставляя в исходную систему, находим $$ z = -150 (-2 + 6t) + 500t = 300-400t. $$ Следовательно, существует бесконечно много решений, которые можно представить в виде множества $$ \ {(x, y, z) \ mid x = -2 + 6t, y = t, z = 300-400t \ text { где $ t \ in \ mathbb {R} $} \}. $

    Другая расширенная матрица для этой системы линейных уравнений — $$ \ left [\ begin {array} {l | l} \ begin {matrix} -150 & 500 & -1 \\ 50 & 100 & 1 \ end {matrix} & \ begin {matrix} 0 \\ 200 \ end {matrix} \ end {array} \ right] $$ Как вы думаете, возможно ли решить, проверяя расширенную матрицу, соответствует ли соответствующая система линейных уравнений будет последовательным или непоследовательным?

    Операции со строками: решение систем линейных уравнений

    До сих пор в наших примерах мы видели системы линейных уравнений, не имеющие решений, единственное решение или, возможно, бесконечное количество решений.Эти примеры предлагают следующее определение.

    Определение . Две системы линейных уравнений называются эквивалентными , если они имеют один и тот же набор решений.

    Пример . Найдите систему линейных уравнений с тремя неизвестными $ x, y, z $, решениями которой являются $ x = 6 + 5t $, $ y = 4 + 3t $, $ z = 2 + t $, где $ t $ произвольно. Мы хотим исключить $ t. $ Решение для $ t $ дает $ t = z-2. $ Путем подстановки $ x = 6 + 5 (z-2) $ и $ y = 4 + 3 (z-2). $ Таким образом, у нас есть система линейных уравнений $$ \ begin {ases} x-5z = -4 \\ y-3z = -2 \ end {ases} $$, которая имеет бесконечно много решений.

    Теорема . Системы линейных уравнений эквивалентны, если каждая может быть получена из другой с помощью одной или нескольких следующих операций.

    Поменяйте порядок уравнений.

    Умножьте (или разделите) одно уравнение на ненулевой скаляр.

    Складываем одно уравнение, кратное одному, к другому.

    Проба . Если мы рассматриваем набор решений как геометрический объект, должно быть очень легко понять, что способ, которым мы пишем уравнения, представляющие объект, не меняет объект.Таким образом, должно быть очевидно, что изменение порядка уравнений не изменит решений системы линейных уравнений. Также не будет умножения (или деления) обеих частей уравнения на ненулевую константу.

    Пусть система $ A $ будет системой $ m \ times n $, представленной \ begin {уравнением} \ label {syseqA} \ begin {cases} a_ {11} x_1 + a_ {12} x_2 + \ cdots + a_ {1n} x_n = b_1 \\ \ hfill \ vdots \ hfill \\ a_ {i1} x_1 + a_ {i2} x_2 + \ cdots + a_ {in} x_n = b_i \\ \ hfill \ vdots \ hfill \\ a_ {m1} x_1 + a_ {m2} x_2 + \ cdots + a_ {mn} x_n = b_m & \\ \ end {cases} \ end {equal} Рассмотрим систему $ B $, полученную из системы $ A $ добавлением $ k $ умноженного на уравнение $ i $ к уравнение $ j $ следующим образом: \ begin {Equation} \ label {syseqB} \ begin {cases} a_ {11} x_1 + a_ {12} x_2 + \ cdots + a_ {1n} x_n = b_1 \\ \ qquad \ qquad \ vdots \ qquad \ qquad \ qquad \ vdots \\ (a_ {j1} + k a_ {i1}) x_1 + (a_ {j2} + k a_ {i2}) x_2 + \ cdots + (a_ {jn} + k a_ {in }) x_n = b_j + k b_i \\ \ qquad \ qquad \ vdots \ qquad \ qquad \ qquad \ vdots \\ a_ {m1} x_1 + a_ {m2} x_2 + \ cdots + a_ {mn} x_n = b_m \\ \ end {case} \ end {Equation} Пусть $ S_A $ и $ S_B $ — множества решений систем $ A $ и $ B $ соответственно.Мы покажем $ S_A = S_B. $

    Пусть $ x_0 = (x_1, x_2, \ ldots, x_n) $ — решение системы $ A. $ Таким образом, $ x_0 $ удовлетворяет всем линейным уравнениям. Итак, $ x_0 $ удовлетворяет всем уравнениям в системе B, кроме, возможно, $ j $ -го уравнения. Работая с $ j $ -м уравнением в системе $ B $, находим

    \ begin {align} & (a_ {j1} + k a_ {i1}) x_1 + (a_ {j2} + k a_ {i2}) x_2 + \ cdots + (a_ {jn} + k a_ {in}) x_n \ \ & \ qquad = a_ {j1} x_1 + \ cdots + a_ {jn} x_n + k (a_ {i1} x_1 + \ cdots a_ {jn} x_n) \\ & \ qquad = b_j + k b_i.\ end {align}

    Это показывает, что $ x_0 $ также удовлетворяет $ j $ -му уравнению системы $ B. $ Поскольку $ x_0 $ удовлетворяет каждому уравнению, $ x_0 $ также является членом $ S_B. $ До сих пор мы показали $ S_A \ substeq S_B . $ И наоборот, предположим, что $ x_0 $ является решением каждого уравнения второй системы линейных уравнений. Работая с $ j $ -м уравнением системы $ A $, находим,

    \ begin {align} & a_ {j1} x_1 + a_ {j2} x_2 + \ cdots + a_ {jn} x_n = b_j \ notag \\ \ Longleftrightarrow \ quad & a_ {j1} x_1 + (k a_ {i1} -ka_ {i1}) x_1 + \ cdots + a_ {jn} + (k a_ {i1} -ka_ {i1}) x_n = b_j \ notag \\ \ Longleftrightarrow \ quad & (a_ {j1} + k a_ {i1}) x_1 + (a_ {j2} + k a_ {i2}) x_2 + \ cdots + (a_ {jn} + k a_ {in}) x_n = b_j + k b_i.\ label {eqsys} \ end {align}

    Следовательно, поскольку $ x_0 $ удовлетворяет системе $ B $; и, таким образом, $ x_0 $ удовлетворяет $ j $ -му уравнению системы $ A. $ Следовательно, $ S_A = S_B $, как и нужно.

    Прорабатывая детали следующих двух примеров, обратите внимание, как используются операции в Row Operations.

    Пример . Пусть $ a, b $ и $ c $ — константы. Решите систему линейных уравнений $$ \ begin {cases} y + z = a \\ x + z = b \\ x + y = c. \ end {ases} $$ Исключая $ z $ из первого и второго уравнений, получаем систему $$ \ begin {ases} xy = ba \\ x + y = c \ end {ases} $$ Решение для $ x $ дает $ x = (c + ba) / 2.$ Используя исходную систему, мы находим, что $ y $ и $ z $ равны $$ y = cx = c- (c + ba) / 2 = (c-b + a) / 2 $$ $$ z = ay = a — (c-b + a) / 2 = (a + bc) / 2 $$ Следовательно, решение системы: $$ (x, y, z) = \ left (\ frac {c + ba} {2 }, \ frac {a + cb} {2}, \ frac {a + bc} {2} \ right). $

    Пример . Найдите наименьшее натуральное число $ C $ такое, что $ x, y, z $ являются целыми числами и удовлетворяют системе линейных уравнений уравнений $$ \ begin {cases} 2x + y = C \\ 3y + z = C \\ x + 4z = C \ end {cases} $$ Умножьте третье уравнение на $ -2 $ и добавьте к первому уравнению, получив $ y-8z = -C.$ Умножая второе уравнение на 8 и прибавляя к $ y-8z = -C $, получаем $ 25y = 7C. $ Решая для $ y $, получаем $ y = (7/25) C. $ Подстановкой $ x = (9/25) C $ и $ z = (4/25) C. $ Следовательно, 25 — наименьшее целое число $ C $ такое, что $ x, y, z $ являются целыми числами и решает систему.

    Исключение по Гауссу

    Мы будем решать систему линейных уравнений, используя элементарные операции со строками над матрицами, используя процедуру, известную как Исключение Гаусса . Набор решений будет набором векторов.

    Определение . Следующие операции в совокупности известны как операции элементарных строк .

    (1) Поменяйте местами два ряда.

    (2) Умножьте строку на ненулевой скаляр.

    (3) Добавить строку из другой строки, кратную строке.

    Из операций со строками очевидно, что применение элементарных операций со строками к системе линейных уравнений приводит к эквивалентной системе. Редукция системы линейных уравнений при сохранении множества решений — чрезвычайно полезная идея, которую мы будем активно развивать.

    Например, мы обнаруживаем, что система линейных уравнений \ begin {equal} \ label {refex} \ begin {cases} x + 2y + z = 3 \\ 2x + 5y-z = -4 \\ 3x-2y- z = 5 \ end {case} \ qquad \ qquad \ begin {cases} x = 2 \\ y = -1 \\ z = 3 \ end {case} \ end {уравнение} эквивалентны. Мы покажем, как применить элементарные операции со строками, чтобы получить систему справа. В то время как система слева может быть заданной системой линейных уравнений, система справа решена.

    Определение .Считается, что матрица находится в эшелоне строки формы , если она удовлетворяет всем следующим условиям.

    (1) Все строки с хотя бы одним ненулевым коэффициентом находятся над любыми строками со всеми нулями.

    (2) Первое ненулевое число слева (называемое ведущим коэффициентом ) ненулевой строки всегда находится строго справа от ведущего коэффициента строки над ней.

    (3) Все записи в столбце под ведущим коэффициентом нулевые.

    Далее, матрица называется сокращенной формой эшелона строк , если она находится в форме эшелона строк и выполняется дополнительное условие: каждый ведущий коэффициент равен 1 и является единственной ненулевой записью в своем столбце.

    Например, расширенные матрицы для системы линейных уравнений выше: $$ \ begin {bmatrix} \ begin {array} {ccc | c} 1 & 2 & 1 & 3 \\ 2 & 5 & -1 & -4 \\ 3 & -2 & -1 & 5 \ end {array} \ end {bmatrix} \ qquad \ qquad \ begin {bmatrix} \ begin {array} {ccc | c} 1 & 0 & 0 & 2 \\ 0 & 1 & 0 & -1 \\ 0 & 0 & 1 & 3 \ end {array} \ end {bmatrix}.$$ Проверив эти четыре условия, мы увидим, что матрица справа имеет вид сокращенного ряда строк. Для получения дополнительных примеров рассмотрим следующие матрицы. $$ A = \ begin {bmatrix} 0 & 5 \\ 2 & 3 \ end {bmatrix} \ qquad B = \ begin {bmatrix} 1 & 0 \\ 1 & 0 \ end {bmatrix} \ qquad C = \ begin {bmatrix} 0 & 0 \\ 0 & 1 \ end {bmatrix} $$ $$ D = \ begin {bmatrix} 0 & 1 \\ 0 & 0 \ end {bmatrix} \ qquad E = \ begin {bmatrix} 1 & 0 \\ 0 & 1 \ end {bmatrix} \ qquad F = \ begin {bmatrix} 1 & 2 \\ 0 & 1 \ end {bmatrix} $$ Обратите внимание, что матрицы $ D $ и $ E $ находятся в уменьшенной строке форма эшелона, а остальные нет.

    Пример . Используйте метод исключения Гаусса-Жордана для решения системы линейных уравнений. $$ \ begin {cases} x + 2y + z = 3 \\ 2x + 5y-z = -4 \\ 3x-2y-z = 5 \ end {cases} $$ Используя операции со строками в расширенной матрице, мы получаем уменьшенная форма рядного эшелона.

    \ begin {align *} & \ begin {bmatrix} \ begin {array} {ccc | c} 1 и 2 и 1 и 3 \\ 2 и 5 и -1 и -4 \\ 3 и -2 и -1 и 5 \ end {массив} \ end {bmatrix} \ begin {array} {c} \ stackrel {\ longrightarrow} {\ scriptstyle -2R_1 + R_2} \\ \ stackrel {\ longrightarrow} {\ scriptstyle -3R_1 + R_3} \ end {массив} \ begin {bmatrix} \ begin {array} {ccc | c} 1 и 2 и 1 и 3 \\ 0 & 1 & -3 & -10 \\ 0 & -8 & -4 & -4 \ end {массив} \ end {bmatrix} \\ & \ qquad \ qquad \ begin {array} {c} \ stackrel {\ longrightarrow} {\ scriptstyle 8 R_2 + R_3} \ end {массив} \ begin {bmatrix} \ begin {array} {ccc | c} 1 и 2 и 1 и 3 \\ 0 & 1 & -3 & -10 \\ 0 & 0 & -28 & -84 \ end {массив} \ end {bmatrix} \ begin {array} {c} \ stackrel {\ longrightarrow} {\ scriptstyle — \ frac {1} {28} R_3} \ end {массив} \ begin {bmatrix} \ begin {array} {ccc | c} 1 и 2 и 1 и 3 \\ 0 & 1 & -3 & -10 \\ 0 и 0 и 1 и 3 \ end {массив} \ end {bmatrix} \\ & \ qquad \ qquad \ begin {array} {c} \ stackrel {\ longrightarrow} {\ scriptstyle 3R_3 + R_2} \\ \ stackrel {\ longrightarrow} {\ scriptstyle -R_3 + R_1} \ end {массив} \ begin {bmatrix} \ begin {array} {ccc | c} 1 и 2 и 0 и 0 \\ 0 & 1 & 0 & -1 \\ 0 и 0 и 1 и 3 \ end {массив} \ end {bmatrix} \ begin {array} {c} \ stackrel {\ longrightarrow} {\ scriptstyle -2R_2 + R_1} \ end {массив} \ begin {bmatrix} \ begin {array} {ccc | c} 1 & 0 & 0 & 2 \\ 0 & 1 & 0 & -1 \\ 0 и 0 и 1 и 3 \ end {массив} \ end {bmatrix}.\ end {align *}

    Следовательно, единственное решение — $ x = 2 $, $ y = -1 $ и $ z = 3. $

    Пример . Используйте метод исключения Гаусса-Жордана для решения следующей системы линейных уравнений. \ begin {уравнение} \ label {gjee1} \ begin {cases} x + y -2z + 4t = 5 \\ 2x + 2y-3z + t = 3 \\ 3x + 3y-4z-2t = 1 \ end {case } \ end {уравнение} Используя операции со строками над расширенной матрицей, мы получаем сокращенную форму эшелона строк.

    \ begin {align *} \ begin {bmatrix} \ begin {array} {cccc | c} 1 и 1 и -2 и 4 и 5 \\ 2 и 2 и -3 и 1 и 3 \\ 3 и 3 и -4 и -2 и 1 \ end {массив} \ end {bmatrix} & \ begin {array} {c} \ stackrel {\ longrightarrow} {\ scriptstyle -2R_1 + R_2} \\ \ stackrel {\ longrightarrow} {\ scriptstyle -3R_1 + R_3} \ end {массив} \ begin {bmatrix} \ begin {array} {cccc | c} 1 и 1 и -2 и 4 и 5 \\ 0 & 0 & 1 & -7 & -7 \\ 0 & 0 & 2 & -14 & -14 \ end {массив} \ end {bmatrix} \\ & \ begin {array} {c} \ stackrel {\ longrightarrow} {\ scriptstyle -2R_2 + R_3} \\ \ stackrel {\ longrightarrow} {\ scriptstyle 2R_2 + R_1} \ end {массив} \ begin {bmatrix} \ begin {array} {cccc | c} 1 и 1 и 0 и -10 и -9 \\ 0 & 0 & 1 & -7 & -7 \\ 0 & 0 & 0 & 0 & 0 \ end {массив} \ end {bmatrix} \ end {align *}

    Эта система эквивалентна $$ \ begin {cases} x + y-10t = -9 \\ z-7t = -7 \ end {ases} \ quad \ text {или, проще говоря} \ quad \ begin {cases} х = -9-у + 10т \\ г = -7 + 7т.4 \ mid x = -9-y + 10t, y = s, z = -7 + 7t, w = t, \ text {for} s, t \ in \ mathbb {R} \}. $$ — это набор решений.

    Пример . Используйте метод исключения Гаусса-Жордана для решения следующей системы линейных уравнений. $$ \ begin {case} x_1 + x_2-2x_3 + 3x_4 = 4 \\ 2x_1 + 3x_2 + 3x_3-x_4 = 3 \\ 5 x_1 + 7 x_2 + 4 x_3 + x_4 = 5 \ end {cases} $$ Использование строки операций над расширенной матрицей мы получаем приведенную форму эшелона строк.

    \ begin {align *} \ begin {bmatrix} \ begin {array} {cccc | c} 1 и 1 и -2 и 3 и 4 \\ 2 и 3 и 3 и -1 и 3 \\ 5 и 7 и 4 и 1 и 5 \ end {массив} \ end {bmatrix} & \ begin {array} {c} \ stackrel {\ longrightarrow} {\ scriptstyle -2R_1 + R_2} \\ \ stackrel {\ longrightarrow} {\ scriptstyle -5R_1 + R_3} \ end {массив} \ begin {bmatrix} \ begin {array} {cccc | c} 1 и 1 и -2 и 3 и 4 \\ 0 и 1 и 7 и -7 и -5 \\ 0 и 2 и 14 и -14 и 15 \ end {массив} \ end {bmatrix} \\ & \ begin {массив} {c} \ stackrel {\ longrightarrow} {\ scriptstyle -2R_2 + R_3} \ end {массив} \ begin {bmatrix} \ begin {array} {cccc | c} 1 и 1 и -2 и 3 и 4 \\ 0 и 1 и 7 и -7 и -5 \\ 0 & 0 & 0 & 0 & 25 \ end {массив} \ end {bmatrix} \ end {align *}

    Обратите внимание, что последняя строка соответствует уравнению $ 0x_1 + 0x_2 + 0x_3 + 0x_4 = 25.2 $ — такой многочлен. Используя данные точки, мы настраиваем систему и решаем.

    \ begin {align *} & \ left [\ begin {array} {c | c} \ begin {matrix} а & б & с \\ а & 2b & 4c \\ a, 3b и 9c \ end {matrix} & \ begin {matrix} -1 \ 3 \ 13 \ end {матрица} \ end {array} \ right] \ begin {array} {c} \ stackrel {\ longrightarrow} {\ scriptstyle -R_1 + R_2} \\ \ stackrel {\ longrightarrow} {\ scriptstyle -R_1 + R_3} \ end {массив} \ left [\ begin {array} {c | c} \ begin {matrix} а & б & с \\ 0 & b & 3c \\ 0 и 2b и 8c \ end {matrix} & \ begin {matrix} 1 \ 4 \ 14 \ end {матрица} \ end {array} \ right] \\ & \ qquad \ qquad \ begin {array} {c} \ stackrel {\ longrightarrow} {\ scriptstyle -2R_2 + R_3} \ end {массив} \ left [\ begin {array} {c | c} \ begin {matrix} а & b & c \\ 0 & b & 3c \\ 0 и 0 и 2c \ end {matrix} & \ begin {matrix} -1 \ 4 \ 6 \ end {matrix} \ end {array} \ right] \ begin {array} {c} \ stackrel {\ longrightarrow} {\ scriptstyle \ frac {1} {2} R_3} \ end {массив} \ left [\ begin {array} {c | c} \ begin {matrix} а & b & c \\ 0 & b & 3c \\ 0 & 0 & c \ end {matrix} & \ begin {matrix} -1 \ 4 \ 3 \ end {матрица} \ end {array} \ right] \\ & \ qquad \ qquad \ begin {array} {c} \ stackrel {\ longrightarrow} {\ scriptstyle -3R_3 + R_2} \ end {массив} \ left [\ begin {array} {c | c} \ begin {matrix} а & б & с \\ 0 & b & 0 \\ 0 & 0 & c \ end {matrix} & \ begin {matrix} -1 \ -5 \ 3 \ end {матрица} \ end {array} \ right] \ begin {array} {c} \ stackrel {\ longrightarrow} {\ scriptstyle -R_2 + R_1} \ end {массив} \ left [\ begin {array} {c | c} \ begin {matrix} а & 0 & с \\ 0 & b & 0 \\ 0 & 0 & c \ end {matrix} & \ begin {matrix} 4 \ -5 \ 3 \ end {матрица} \ end {array} \ right] \\ & \ qquad \ qquad \ begin {array} {c} \ stackrel {\ longrightarrow} {\ scriptstyle -R_3 + R_1} \ end {массив} \ left [\ begin {array} {c | c} \ begin {matrix} а & 0 & 0 \\ 0 & b & 0 \\ 0 & 0 & c \ end {matrix} & \ begin {matrix} 1 \ -5 \ 3 \ end {матрица} \ end {array} \ right].2-5 лет + 1. $

    Упражнения по системе линейных уравнений

    Упражнение . Убедитесь, что $ (2,3, -1) $ является решением системы линейных уравнений.

    • $ \ begin {cases} x + 2y + z = 7 \\ xy = -1 \\ 4x + y + z = 10 \ end {cases} $
    • $ \ begin {cases} x + y = 5 \ \ xz = 3 \\ y + z = 2 \ end {cases} $

    Упражнение . Убедитесь, что каждая тройка вида $ (7-2k, 8 + 6k, k) $ является решением системы линейных уравнений.

    • $ \ begin {cases} x_1 + 2x_3 = 7 \\ x_2-6x_3 = 8 \ end {cases} $
    • $ \ begin {cases} 2x_1 + 4x_3 = 14 \\ x_1 + 3x_2-16x_3 = 31 \ end {кейсы}

    Упражнение .У следующих систем нет решений, ровно одно решение или бесконечно много решений? Обосновать ответ.

    • $ \ begin {case} x + y = 1 \\ x + 2y = 1 \ end {cases} $
    • $ \ begin {cases} 3x + y = 1 \\ y = -2 \ end {cases} $
    • $ \ begin {case} x + y = 1 \\ 2x + 2y = 2 \ end {cases} $
    • $ \ begin {cases} 3x + y = 2 \\ 6x + 2y = 4 \ end { case} $

    Упражнение . Нарисуйте график каждого уравнения системы линейных уравнений и решите, нет ли у него решений, ровно одно решение или бесконечно много решений.

    • $ \ begin {case} x + y = 2 \\ 2x + 3y = 0 \ end {cases} $
    • $ \ begin {cases} -x + 3y = 2 \\ 2x-6y = -4 \ end {кейсы}

    Упражнение . Найдите решения, если таковые имеются, следующей системы линейных уравнений без использования матриц. Если уравнение имеет более одного решения, напишите общее решение.

    • $ \ begin {cases} 3x-2y = 7 \\ 2x + y = 15 \ end {cases} $
    • $ \ begin {cases} x + y = 7 \\ 3x + 4y = 12 \ end {случаях } $
    • $ \ begin {case} 2x_1-3x_2 = 1 \\ 4x_1-6x_2 = -2 \\ x_1 + x_2 = 1 \ end {cases} $
    • $ \ begin {cases} 2x_1-5x_2 = 12 \ end {case} $
    • $ \ begin {cases} i x_1-3ix_2 = 1 \\ (2 + i) x_1-x_2 = -1 \ end {cases} $
    • $ \ begin {cases} (1 + i ) x_1-2ix_2 = 2 \\ 2x_1-3i x_2 = 4-3i \ end {cases} $

    Упражнение .Если возможно, найдите точки пересечения.

    • $ x-4y = 11 $ и 7x-2y = 9 $
    • $ x-4y + 3z = 11 $, 7x-2y-z = -1 $, 7x-2y + z = -2 $

    Упражнение . Пусть $ u = (1,1,2, -1) $ и $ v = (1,1,1,0). $ Для каких скаляров $ a $ и $ b $ верно, что $ a u + bv $ это решение следующей системы?

    • $ \ begin {cases} 4x-2y-zw = 1 \\ x + 3y-2z-2w = 2 \ end {ases} $
    • $ \ begin {cases} x-4y-z-2w = 4 \ \ 7x + y-5z-2w = 12 \ end {cases} $

    Упражнение .Найдите все решения, если таковые существуют, для следующей системы линейных уравнений.

    • $ \ begin {case} xy = 1 \\ 2x = 4 \ end {cases} $
    • $ \ begin {cases} 2x + 3y-z = 19 \\ 3x-2y + 3z = 7 \ end {случаях } $
    • $ \ begin {case} x-3y = 2 \\ -2x + 6y = -4 \ end {cases} $
    • $ \ begin {cases} x-y + 2z-2w = 1 \\ 2x + y + 3w = 4 \\ 2x + y + 3w = 6 \ end {cases} $

    Упражнение . Найдите все решения, если таковые существуют, следующей системы линейных уравнений.

    • $ \ begin {cases} 2x-z = -1 \\ x + yz = 0 \\ 2x-y + 2z = 3 \ end {cases} $
    • $ \ begin {cases} 2x-3y + 2z = 1 \\ x-6y + z = 2 \\ -x-3y-z = 1 \ end {cases} $

    Упражнение .Найдите все решения, если таковые существуют, следующей системы линейных уравнений.

    • $ \ begin {cases} x_1 + 2x_2-x_3 = 0 \\ 2x_1 + 5x_2 + 5x_3 = 0 \\ x_1 + 4x_2 + 7x_3 = 0 \\ x_1 + 3x_2 + 3x_3 = 0 \ end {cases} $
    • $ \ begin {case} x + y + z + w = ​​1 \\ 2x-2y + z + 2w = 3 \\ 2x + 6y + 3z + 2w = 1 \\ 5x-3y + 3z + 5w = 8 \ end {кейсы}

    Упражнение . При каких значениях константы $ k $ у систем нет решения, ровно одно решение или бесконечное множество решений.

    • $ \ begin {cases} x + y = 1 \\ 3x + 3y = k \ end {cases} $
    • $ \ begin {cases} x + ky = 1 \\ 2x-y = k \ end {случаях } $

    Exercise .2 + 2} + \ frac {c} {2x-1}. $

    Упражнение . Пусть $ a $ и $ b $ — произвольные постоянные. Найдите все решения системы линейных уравнений.

    • $ \ begin {cases} x + 2y = a \\ 3x + 5y = b \ end {cases} $
    • $ \ begin {cases} a x + 2y = 1 \\ 3x + by = 4 \ end { case} $

    Упражнение .
    Пусть $ a $ и $ b $ — произвольные постоянные. Найдите все решения системы линейных уравнений.

    • $ \ begin {case} x + 2y + 3z = a \\ x + 3y + 8z = b \\ x + 2y + 2z = c \ end {case} $
    • $ \ begin {cases} x-2y + 4z = a \\ x-3y + 5z = b \\ x-2y + 6z = c \ end {cases} $

    Упражнение .Система линейных уравнений, все постоянные члены которой равны нулю, называется однородной системой
    .

    • Покажите, что в однородной системе всегда есть хотя бы одно решение.
    • Приведите примеры, показывающие, что гомогенная система может иметь более одного решения или только одно решение.

    Упражнение . Напишите систему, соответствующую каждой расширенной матрице.

    • $ \ left [\ begin {array} {ccc | c} 1 & 2 & 1 & 1 \\ 0 & 4 & 0 & 1 \\ 0 & 0 & 3 & 3 \ end {array} \ right] $
    • $ \ left [\ begin {array} {cccc | c} 1 & 0 & 0 & 0 & 0 \ end {array} \ right] $
    • $ \ left [\ begin {array} {ccc | c} 1 & 0 & 0 & 0 \\ 0 & 1 & 0 & 0 \\ 0 & 0 & 1 & 0 \ end {array} \ right] $
    • $ \ left [\ begin {array} {c | c} 2 & 1 \\ 0 & 1 \\ 1 & 2 \ end {array} \ right] $
    • $ \ left [\ begin {array} {c | c} 1 & 1 \\ 1 & 0 \ end {массив } \ right] $
    • $ \ left [\ begin {array} {cccc | c} 1 & 1 & 1 & 1 & 0 \ end {array} \ right] $

    Упражнение .Какие из следующих матриц представлены в виде уменьшенного ряда строк?

    • $ \ begin {bmatrix} 0 & 1 & 2 & 0 & 3 \\ 0 & 0 & 0 & 1 & 0 \\ 0 & 0 & 0 & 0 & 0 & 0 \ end {bmatrix} $
    • $ \ begin {bmatrix} 1 & 0 & 0 & 2 & 1 \\ 0 & 1 & 0 & 0 & 0 \\ 0 & 0 & 1 & 0 & 0 \ end {bmatrix} $
    • $ \ begin {bmatrix} 1 & 1 & 1 \\ 0 & 0 & 1 \\ 0 & 0 & 0 \\ 0 & 0 & 0 \ end {bmatrix} $
    • $ \ begin {bmatrix} 0 & 0 & 1 \\ 0 & 0 & 1 \\ 0 & 0 & 1 \\ \ end {bmatrix} $

    Упражнение .Какие из следующих матриц представлены в виде уменьшенного ряда строк?

    • $ \ begin {bmatrix} 0 & 0 & 0 & 1 \\ 0 & 0 & 0 & 0 \\ \ end {bmatrix} $
    • $ \ begin {bmatrix} 0 & 0 & 0 \\ 0 & 0 & 0 \\ \ end {bmatrix} $
    • $ \ begin {bmatrix} 1 & 0 & 0 & 3 & 1 \\ 0 & 0 & 0 & 1 & 1 & 1 \\ 0 & 0 & 0 & 0 & 1 \ end {bmatrix} $
    • $ \ begin {bmatrix} 1 & 0 & 2 & 1 \\ 0 & 1 & 0 & 1 \\ 0 & 0 & 1 & 0 \\ 0 & 0 & 0 & 0 \ end {bmatrix } $

    Exercise .Найти все решения системы линейных уравнений методом исключения Гаусса-Жордана.

    • $ \ begin {cases} x_2 + 2x_4 + 3x_5 = 0 \\ 4x_4 + 8x_5 = 0 \ end {cases} $
    • $ \ begin {cases} 3x + 4y = 0 \\ -2x + 7y = 0 \ конец {кейсы} $

    Упражнение . Найти все решения системы линейных уравнений методом исключения Гаусса-Жордана.

    • $ \ begin {cases} x_4 + 2x_5-x_6 = 2 \\ x_1 + 2x_2 + x_5-x_6 = 0 \\ x_1 + 2x_2 + 2x_3-x_5 + x_6 = 2 \ end {cases} $
    • $ \ begin {case} 3x_1 + 3x_2-4x_3 + x_4 = 2 \\ x_1 + x_2-x_3-x_4 = 5 \ end {cases} $
    • $ \ begin {cases} 2 x_1 + 3x_2 + 4x_3 = 6-x_4 \\ 3x_1 -2x_2-x_4 = 1 + 4x_3 \\ 3x_1 + 3x_3 + x_4 = 4-x_2 \\ x_2 + x_3-4x_4 = -3-4x_1 \ end {cases} $
    • $ \ begin {cases} 4x_1 + 3x_2 + 2x_3 -x_4 = 4 \\ 5x_1 + 4x_2 + 3x_3-x_4 = 4 \\ -2x_1-2x_2-x_3 + 2x_4 = -3 \\ 11x_1 + 6x_2 + 4x_3 + x_4 = 11 \ end {case} $
    • $ \ begin {case} 2x_1-3x_2 + x_3 = 5 \\ x_1 + x_2-x_3 = 3 \\ 4x_1-x_2-x_3 = 1 \ end {cases} $
    • $ \ begin {cases} x_1-x_2 = 4 \\ 2x_1 + x_2 = 7 \\ 5x_1-2x_2 = 19 \ end {case} $

    Упражнение .Найдите все матрицы размером $ 4 \ times 1 $ в приведенной форме эшелона строк.

    Упражнение . Сколько существует типов матриц размером $ 3 \ times 2 $ в приведенной строчной форме? Сколько существует типов матриц размером $ 2 \ times 3 $ в приведенной строчной форме?

    Упражнение . Опишите возможные формы сокращенного эшелона строк для матрицы с двумя строками и двумя столбцами. Опишите возможные формы сокращенного эшелона строк для матрицы с тремя строками и тремя столбцами.

    Упражнение .Найдите многочлен степени 3, график которого проходит через точки
    $ (0,1) $, $ (1,0) $, $ (- 1,0) $ и $ (2, -15). $ Нарисуйте эскиз график этой кубики.

    Упражнение . Найдите многочлен степени 4, график которого проходит через точки $ (1,1) $, $ (2, -1) $, $ (3, -59) $ и $ (- 2, -29). $ Sketch график этой квартики.

    Упражнение . Найдите значения $ k $, если они есть, для которых система имеет: только одно решение, никаких решений, бесконечное количество решений.2-2) x_3 = a \ end {cases} $$ будет иметь: бесконечно много решений, никаких решений, ровно одно решение.

    Упражнение . Используйте метод исключения Гаусса-Жордана для решения системы линейных уравнений $$ \ left \ {\ begin {array} {lll} a x_1 + b x_2 & & = r \ quad (a \ neq 0) \\ c x_1 + d x_2 & & = s \ quad (e \ neq 0) \\ & e x_3 + f x_4 & = t \\ & g x_3 + h x_4 & = u \\ \ end {array} \ right. $$ Условия состояния для $ a, b, c, d, e, f, g $ и $ h $, которые гарантируют уникальное решение.2 + a x + by + c = 0 $ окружности, проходящей через следующие точки.

    • $ (- 2,1) $, (5,0) $ и (4,1) $
    • $ (1,1) $, $ (5, -3) $ и $ (- 3, -3) $

    Часть 6: Исключение по Гауссу. Исключение Гаусса — это алгоритм… | Авниш | Линейная алгебра

    Метод исключения Гаусса — это алгоритм решения системы линейных уравнений. Он назван в честь немецкого математика Карла Фридриха Гаусса.

    Карл Фридрих Гаусс

    Он аналогичен методу исключения, описанному ранее.

    Для выполнения исключения Гаусса:

    1. Создаем расширенную матрицу коэффициентов и констант данной системы линейных уравнений.
    2. Выбираем наш pivot (который является первым элементом по диагонали). Затем мы пытаемся уменьшить все элементы под ним (до «0»), используя pivot.

    Мы делаем это, выполняя два вида операций:

    a) Умножение сводной строки (строки сводного элемента) на скалярную величину и вычитание или добавление ее строк под ней.

    b) Перестановка строк (например, строка 2 заменяется строкой 3)

    Затем мы выбираем следующую точку поворота (следующий элемент по диагонали) и уменьшаем элементы под ней.

    3. Разбиваем расширенную матрицу обратно на строковое изображение и выполняем умножение с переменной матрицей. Получаем новые редуцированные уравнения.

    Мы решаем эти уравнения, чтобы получить значения неизвестных (переменных).

    Предположим, что нам нужно найти решение (я) следующей системы уравнений:

    4x + y = 9 → (1)

    2x-y = 3 → (2)

    5x-3y = 7 → ( 3)

    (пример «Одно уникальное решение» из Части 5)

    Шаг 1 (Создание расширенной матрицы):

    Для выполнения исключения Гаусса мы берем изображение строки (1), (2) и (3) .Это будет выглядеть следующим образом:

    Затем мы создаем расширенную матрицу для матрицы коэффициентов и постоянной матрицы.

    Единая матрица со значениями коэффициентов и констант, разделенных пунктирной линией

    Шаг 2 (Исключение):

    Шаг 2A:

    Принимая элемент в верхнем левом углу (первый элемент по диагонали) в качестве стержня, мы стремимся исключить ( уменьшить до «0») все элементы под ним. Другими словами, мы должны преобразовать каждый элемент в столбце 1 в «0», кроме pivot.

    Pivot элемент будет выделен красным цветом, а элементы, которые нужно исключить, — синим.

    Итак, мы умножаем первую строку на скаляр 1/2 и вычитаем ее из второй строки.

    Элемент в строке 2 и столбце 1 исключается.

    Затем мы умножаем первую строку на скаляр 5/4 и вычитаем из третьей строки.

    Элемент в строке 3 и столбце 1 исключен.

    Теперь все элементы в первом столбце равны «0», кроме точки поворота.

    Шаг 2B:

    Теперь следующий элемент по диагонали (второй столбец второй строки) установлен как опорный, и мы стремимся удалить все элементы под ним.

    Pivot выделен красным.

    Итак, мы умножаем вторую строку на скаляр 17/6 и вычитаем ее из третьей строки.

    Элемент в строке 3 и столбце 2 исключен.

    Результат — верхняя треугольная матрица.

    Текущее состояние расширенной матрицы называется эшелоном строк формы .

    Шаг 3 (обратная подстановка):

    Теперь мы конвертируем форму эшелона строки обратно в изображение строки.

    У нас было аналогичное уравнение на этапе 1

    При умножении мы получаем:

    Мы составляем уравнения из этих

    4x + y = 9 → (4)

    -3y / 2 = -3/2 → (5)

    Решая (5) относительно «y», получаем:

    y = 1

    Теперь подставляем y = 1 в (4):

    4x + 1 = 9

    4x = 8

    x = 2

    Итак, мы получаем x = 2 и y = 1, именно то, что мы получили, когда решали через изображение строки и изображение столбца в Части 5.

    Теперь применим тот же алгоритм еще в двух случаях (бесконечно много решений и нет решения).

    Бесконечно много решений

    Возьмем тот же пример, что и в части 5. А именно:

    x + 2y = 4 → (6)

    2x + 4y = 8 → (7)

    Шаг 1 (Создание дополненного матрица):

    Строковое изображение (6) и (7) Расширенная матрица строкового изображения выше

    Шаг 2 (Исключение):

    Первый элемент диагональный («1») в качестве опорного.

    Pivot выделен красным, и мы должны удалить все элементы под ним (синим). Чтобы исключить «2», мы дважды вычитаем строку 1 из строки 2 Теперь последняя строка полностью заполнена 0

    Мы больше не делаем поворота так как исключать нечего.

    Шаг 3 (обратная подстановка):

    Мы конвертируем форму эшелона строки обратно в изображение строки:

    После этого мы умножаем ее и получаем новые уравнения

    x + 2y = 4 → (8)

    Уравнение (6) и уравнения (8) такие же, и у нас есть только одно уравнение после исключения, но два неизвестных («x» и «y»).

    Существует множество значений, которыми можно заменить x и y, чтобы удовлетворить (8).

    Нравится, x = 0 и y = 2. Подставляя в уравнение (8), получаем:

    0 + 2 (2) = 4

    4 = 4

    Или x = 1 и y = 1.5. Подставляя в уравнение (8), получаем:

    1 + 2 (1.5) = 4

    1 + 3 = 4

    4 = 4

    Таким образом, система уравнений (6) и (7) имеет бесконечно много решений.

    Нет решения

    Рассмотрим систему линейных уравнений следующим образом:

    x + y = 4 → (9)

    x + y = 8 → (10)

    xy = 0 → (11)

    Применение гауссиана Устранение.

    Шаг 1 (Создание расширенной матрицы):

    Строковое изображение (9), (10) и (11) Расширенная матрица коэффициентов и констант

    Шаг 2 (Исключение):

    Принятие первого диагонального элемента в качестве оси

    Мы выполняем следующие две операции:

    и матрица, которую мы получаем:

    У нас все еще нет формы эшелона строк (верхняя треугольная матрица).

    Итак, мы выполняем обмен строк (который также является вариантом на этапе исключения из метода исключения по Гауссу):

    Замена строки 3 на строку 2 Форма эшелона строк

    Шаг 3 (обратная подстановка):

    Форма эшелона строк преобразована обратно в изображение строки

    Уравнения, которые мы получаем после умножения матриц выше:

    x + y = 4 → (12)

    -2y = -4 → (13)

    Решая уравнение (13) относительно «y», получаем:

    y = 2

    Подставляя y = 2 в уравнение (12), мы получаем:

    x + 2 = 4

    x = 2

    Чтобы подтвердить, что x = 2 и y = 2 является решением, мы подставляем их в систему уравнений i.е. (9), (10) и (11).

    Подставляя в (9), получаем:

    2 + 2 = 4

    4 = 4

    x = 2 и y = 2, удовлетворяет (9).

    Подставляя в (10), получаем:

    2 + 2 = 8

    4 ≠ 8, это не удовлетворяет (10).

    Следовательно, x = 2 и y = 2 не является решением (9), (10) и (11), и не существует решения этой системы линейных уравнений, как мы видели в прошлой статье.

    Одно решение

    Когда количество неизвестных (переменных) равно количеству уравнения в системе линейных уравнений.

    На примере (1), (2) и (3):

    4x + y = 9 → (1)

    2x-y = 3 → (2)

    5x-3y = 7 → (3)

    Есть 2 неизвестных («x» и «y») и 3 уравнения ((1), (2) и (3)).

    Двух уравнений было бы достаточно для двух неизвестных.

    Бесконечно много решений

    Когда количество неизвестных превышает количество уравнений.

    В примере (6) и (7):

    x + 2y = 4 → (6)

    2x + 4y = 8 → (7)

    Есть 2 неизвестных («x» и «y») и 2 уравнения ((6) и (7)).

Добавить комментарий

Ваш адрес email не будет опубликован. Обязательные поля помечены *